Pathophysiology exam 1

Pataasin ang iyong marka sa homework at exams ngayon gamit ang Quizwiz!

Collateral Circulation

A mechanism for long-term regulation of local blood flow. In the heart and other vital structures, anastomotic channels exist between some smaller arteries that permit perfusion of an area by more than one artery. If one artery is occluded, anastomotic channels increase in size, allowing blood from a patent artery to perfuse the area. ● People with extensive obstruction of a coronary blood vessel may rely on collateral circulation to meet the oxygen needs of the myocardial tissue. Two pts in ER, older and younger, the older pt has a higher probability of living longer because of the longer years of compensatory mechanisms.

myocarditis

inflammation of myocardium; most often caused by viral or bacterial infection

endocarditis

inflammation of the inner (lining) of the heart (particularly heart valves)

pericarditis

inflammation of the membrane surrounding the heart, the pericardium

How to identify a heart failure exacerbation

- Fluid retention and edema (increased capillary pressure in peripheral/pulmonary circulation, can have nocturia - waking at night to urinate) - Respiratory manifestation - dyspnea, congestion of pulm circulations, exertional dyspnea (indication of inadequate ventilation of reduced O2 in circulation - Fatigue, weakness, cognitive impairment- diminished output from left ventricle, progress throughout day, anxiety/restlessness/insomnia - Cachexia and malnutrition - fatigue leads to lack of nutrient intake. Congestion of liver leads to impaired metabolism and impaired absorption of nutrients taken in - Cyanosis - desaturated hemoglobin, oral/peripheral areas - Arrhythmias via EKG - Weight gain

Children and increased risk for respiratory disorders, why? (book pg 797, 5th edition)

- Physically smaller and functionally immature - Significant structural differences in child <8 yrs of age, increase risk for respiratory problems - Child's respiratory rate is higher than adults = increased metabolic rate increased O2 need] - Lung size proportional to body weight - Lung volumes proportional to lung size - Lack of surfactant in infants, not strong enough to inflate alveoli - Alveolar development increased until around age 8 - infants/children use ab muscles to inhale until 5-6 yrs - Tongue larger proportional to mouth; tonsillar tissue normally enlarged in school age children - Flexible larynx is more susceptible to spasm - Eustachian tubes relatively horizontal (common for ear infections) - Children are immunocompetent due to lack of exposure, immunity is still being exposed and strengthened - Younger the child, the more likely you'll see alterations in normal respiratory function - Because of the small size of airway in infants & children, respiratory tract infections are often more serious in this group. Infection may cause sore throat/hoarseness in adults, but may produce serious obstruction in child - Respiratory tract infections that affect small children are croup, bacterial tracheitis, epiglottitis (life threatening), and bronchiolitis

Risk Factors for developing heart failure

- Primary risk factors: coronary artery disease (CAD) and HTN * main risk factor (silent killer) - Age, more prevalent in geratric patients - diabetes, - Smoking, - Obesity - Faulty heart valves - Genetics - Drug use esp cocaine and alcohol abuse

Potential outcomes and treatments of abdominal aortic aneurysm

- Risk factor modification - Hypercholesterolemia and high blood pressure should be controlled and smoking discontinued - Surgical Repair: the involved section of the aorta is replaced with a synthetic graft of woven Dacron (frequently the treatment of choice) - Aortic Dissection: hemorrhage into the vessel wall with longitudinal tearing of the vessel wall to form a blood-filled channel

Risk factors—> Abdominal Aortic Aneurysm

- The two common causes of aortic aneurysms are atherosclerosis and degeneration of the vessel media - Most abdominal aneurysms are asymptomatic - Hypercholesterolemia - Half of the people with aortic aneurysms have hypertension - Aortic aneurysms develop more frequently in males after the age of 50 years who smoke cigarettes - Genetic predisposition - This is an emergency situation → go to hospital ASAP

Use of Antiviral medications in viral infections

- Used to interrupt viral replication, and also tend to interfere with host cell reproduction - Decrease ability of viruses to multiply - Antiviral drugs do not inactivate or kill viruses (because then they would likely kill the host cells as well), they merely inhibit viral reproduction by interfering with a certain stage of the virus life cycle Antiviral agents ● Block viral RNA and DNA synthesis ● Block viral binding to cells ● Block the production of new protein coats (capsids) of new viruses ● Make viruses "go to sleep"

Patient education on valve failure?

1. Know the symptoms and talk to your doctor if you have one or more: ● Chest pain ● Shortness of breath ● Lightheaded 2. Symptoms can come on gradually for aging population 3. Left untreated, advanced valve disease can cause heart failure, stroke, blood clots or death due to sudden cardiac arrest. 4. Risk factors include: ● People who have had rheumatic fever or infective endocarditis are at greater risk for heart valve problems. ● People who have heart problems such as heart attack, heart failure, arrhythmia or previous heart valve conditions from birth (congenital heart defects) ● Childhood cancer survivors who had radiation therapy to the chest also have an increased prevalence of heart valve disease later in life.

Atrial natriuretic peptide (ANP) (Patho Textbook 4th ed Chapter 20,page 490 &611)

ANP belongs to a group of natriuretic peptides that are produced and secreted by the heart muscle. Natriuretic peptides have potent diuretic, natriuretic, vascular smooth muscle, and other neurohumoral actions that affect cardiovascular function. ● It is commonly associated with heart failure. ● It is released from atrial cells in response to atrial stretch pressure, or fluid overload. ● Circulating levels of both ANP and BNP(question 35 explain BNP) are elevated in persons with heart failure. The concentrations are well correlated with the extent of ventricular dysfunction, increasing up to 30-fold in persons with advanced heart disease Atrial natriuretic peptide (ANP) is also believed to have an important role in salt and water excretion by the kidney. ● ANP Increased levels of this peptide directly inhibit the reabsorption of sodium and water in the renal tubules. ● ANP also inhibits renin secretion and therefore angiotensin II formation, which in turn reduces reabsorption of sodium. ● The decrease in sodium reabsorption increases urine output and helps return blood volume to normal. ● ANP levels, which become elevated when the atria are stretched in congestive heart failure, help to decrease vascular volume by increasing urine output. ● Video: https://www.youtube.com/watch?v=vOgQnsKd830&t=222s ○ Summary: i. Right Atrium over-stretches (increased preload) due to increased volume and ultimately increased blood pressure ii. Atrial Natriuretic Peptide (ANP) is released by the right atrium iii. ANP causes vasodilation which will decrease blood pressure iv. ANP will increase urine output that will help decrease blood volume and blood pressure

Ischemia and other toxic injuries increase the accumulation of intracellular calcium as a result of: A) Release of stored calcium from the mitochondria B) Improved intracellular volume regulation C) Decreased influx across the cell membrane D) Attraction of calcium to fatty infiltrates

ANS: A) Release of stored calcium from the mitochondria

Replacement/Therapies/Nursing Interventions

AORTIC therapies: - Valve replacement is recommended for people who are symptomatic - Valve replacement is still controversial for patients who are asymptomatic - ACE inhibitors for asymptomatic severe aortic regurgitation (esp if they have HTN) - Use medications that improves stroke volume and reduces regurgitant volume usually through the use of afterload reducers MITRAL therapies: - ACE inhibitors and bi ventricular pacing (pacemaker) - Surgeries: mitral valve repair and replacement with or without removal of the mitral apparatus - Surgeries recommended for severe cases - Mitral valve repair avoids use of anticoagulation needed for artificial valves

ATP depletion signs and symptoms

ATP depletion: - Pathogenesis is related to hypoxia and even ischemia. - Important effect = acute cell swelling caused by failure of energy-dependent sodium/potassium membrane pump. Leading to potassium decrease and sodium and water retention in the cell. Where sodium goes water follows!

A client with poor arterial circulation in the lower limbs has developed areas of inflammation and "weeping" clear serous exudate. Since chronic inflammation lasts for a long time, it has been associated with which of the following changes in physiological response? Select all that apply. A) Formation and development of new blood vessels B) The death of one or more cells in the body within a localized area C) Regulation and modulation of the immune repsonse through synthesis and relapse of inflammatory mediators D) Release of scavenger cells capable of engulfing bacteria through phagocytosis

Ans: A, B A) Formation and development of new blood vessels B) The death of one or more cells in the body within a localized area

A 5-year-old child is experiencing itchy, watery eyes and an increased respiratory rate with some inspiratory wheezes. He has been outside playing in the yard and trees. The mother asks, "Why does he get like this?" The health care worker's best response is: A) "This is what we call a type I hypersensitivity reaction and usually occurs a few minutes after exposure to his allergen. It is primarily caused by mast cells in his body." B) "Because his allergy is related to something in his environment, the best thing you can do is try to keep him indoors as much as possible." C) "This sounds like he is on his way to having an anaphylactic reaction and you need to get a prescription for an EpiPen to decrease his response to monocytes." D) "This is pretty common in children. He is just getting used to all the allergens in the air. I suggest you just give him a shower after every time he plays outside."

Ans: A) "This is what we call a type I hypersensitivity reaction and usually occurs a few minutes after exposure to his allergen. It is primarily caused by mast cells in his body." Feedback: The immediate response to allergen exposure is mast cell degranulation and release of mediators such as histamine and acetylcholine. Monocytes respond as part of the acute immune response. There is no truth to this being an anaphylactic reaction. Showers may help, but the underlying cause is the mast cell degranulation and the release of preformed mediators.

Which of the following clients would be an example of cellular atrophy? A) A middle-aged femal experiencing menopause due to loss of estrogen stimulation B) A postnephrectomy client whose remaining kidney enlarges to compensate for the loss C) A hypertensive, noncompliant client who has developed a progressive increase in left ventricular mass D) A female client with the change in uterine size as a result of pregnancy

Ans: A) A middle-aged femal experiencing menopause due to loss of estrogen stimulation

Which of the following is a nonmodifiable risk factor for the development of primary hypertension? A) African American race B) High salt intake C) Male gender D) Obesity

Ans: A) African American race

Which of the following processes associated with cellular injury is most likely to be reversible? A) Cell damage resulting from accumulation of fat in the cytoplasm B) Cellular changes as a result of ionizing radiation C) Cell damage resulting from accuumulation of free radicals D) Apoptosis

Ans: A) Cell damage resulting from accumulation of fat in the cytoplasm

A client presents for a scheduled Papanicolaou (Pap) smear. The clinician who will interpret the smear will examine cell samples for evidence of: A) Changes in cell shape, size, and organization B) The presence of unexpected cell types C) Ischemic changes in cell samples D) Abnormally high numbers of cells in a specified field

Ans: A) Changes in cell shape, size, and organization

Free radicals damage cells by: A) Destroying phospholipids in the cell membrane B) Altering the immune response of the cell C) Distrupting calcium storage in the cell D) Inactivation of enzymes and mitochondria

Ans: A) Destroying phospholipids in the cell membrane

An 84-year-old client's blood cultures have come back positive, despite the fact that his oral temperature has remained within normal range. Which of the following phenomena underlies the alterations in fever response that occur in the elderly? A) Disturbance in the functioning of the thermoregulatory center B) Increased heat loss by evaporation C) The presence of comorbidities that is associated with lowered core temperature D) Persistent closure of arteriovenous shunts

Ans: A) Disturbance in the functioning of the thermoregulatory center Feedback: The probable mechanisms for the blunted fever response in older adults include a disturbance in sensing of temperature by the thermoregulatory center in the hypothalamus. Heat loss by evaporation tends to decrease with age, and the closure of AV shunts results in increased core temperature. The presence of comorbidities is not noted to contribute to reduced fever response in older adults.

In the ICU setting, clients who develop shock need thorough head-to-toe assessments. Which of the following clinical manifestations would alert the health care provider that client may be developing ischemia associated with gastrointestinal redistribution of blood flow? A) Gastric bleeding B) Nausea and vomiting C) Irritable bowel syndrome D) Copius high-volume diarrhea

Ans: A) Gastric bleeding

Epithelialization, the first component of the proliferative phase of wound healing, is delayed in open wounds until what type of tissue has formed? A) Granulation tissue B) Fibrinous meshwork C) Capillary circulation D) Collagenous layers

Ans: A) Granulation tissue

A client who lives with angina pectoris has taken a sublingual dose of nitroglycerin to treat the chest pain he experiences while mowing his lawn. This drug facilitates release of nitric oxide, which will have what physiologic effect? A) Smooth muscle relaxation of vessels B) Decreased heart rate and increased stroke volume C) Increased preload D) Reduction of cardiac refractory periods

Ans: A) Smooth muscle relaxation of vessels Feedback: Nitroglycerin produces its effects by releasing nitric oxide in vascular smooth muscle of the target tissues, resulting in relaxation of this muscle and increased blood flow. This drug does not decrease heart rate. Because it vasodilates, it decreases preload. Nitroglycerine does not affect cardiac refractory periods.

The plaques in a client's coronary arteries are plentiful, and most have small- to moderate-sized lipid cores with thick fibrous caps. This form of atherosclerosis is most closely associated with which of the following diagnoses? A) Stable angina B) Non-ST-segment elevation MI C) ST-segment elevation MI D) Unstable angina

Ans: A) Stable angina Feedback: The fixed or stable plaque is commonly associated with stable angina, and the unstable plaque is implicated in unstable angina and myocardial infarction (MI)

A male client with a diagnosis of liver cancer has been recently admitted to a palliative care unit following his recent development of bone metastases. His family shares with the nurse that even though he is usually a "big eater," he just isn't interested in food. This has resulted in a loss of muscle mass. Which of the following factors may underlie the client's change in nutritional status? A) The action of cytokines and persistent inflammation B) Loss of appetite due to fatigue and pain C) Changes in peptide hormone levels D) Production of onconeural antigens by cancerous cells

Ans: A) The action of cytokines and persistent inflammation

A client has been brought to the emergency department with a large, gaping, wound from a farming accident. The client is critically ill and has required blood products and surgery to clean and close the wound. Sharing with a student nurse the steps in wound healing, the nurse discusses the inflammatory phase, stating macrophages: Select all that apply. A) Release growth factors that stimulate epithelial cell growth B) Facilitate the body's ability to grow new vessels in the injured area C) Remove debris from the wound D) Synthesize and secrete collagen E) Proliferate to form the granulation tissue to serve as a foundation for scar tissue to form

Ans: A, B, C A) Release growth factors that stimulate epithelial cell growth B) Facilitate the body's ability to grow new vessels in the injured area C) Remove debris from the wound

The nurse notes the client has developed a systemic response of inflammation based on assessment findings. Which of the following clinical manifestations support this diagnosis? Select all that apply. A) Temperature of 100.9 F, lethargy B) Pulse rate 130 beats/minute (high) C) Generalized achiness D) Low urine output E) Pounding, throbbing headache

Ans: A, B, C A) Temperature of 100.9 F, lethargy B) Pulse rate 130 beats/minute (high) C) Generalized achiness

A client with a history of antiphospholipid syndrome presents to the emergency department. Which of the following complaints lead the nurse to suspect the client is experiencing a complication related to the syndrome? Select all that apply. A) Swelling in the right calf with redness and tenderness B) Having respiratory symptoms like shortness of breath and chest pain C) Have developed a cough that produced some bloody sputum D) Developed a headache that has lasted for 3 days now E) Having trouble making "water" and has pain in the lower abdomen and groin

Ans: A,B,C A) Swelling in the right calf with redness and tenderness B) Having respiratory symptoms like shortness of breath and chest pain C) Have developed a cough that produced some bloody sputum Feedback: Persons with the disorder present with a variety of clinical manifestations, typically those characterized by recurrent venous and arterial thrombi. Venous thrombosis (DVT), especially in the deep leg veins, occurs in up to 50% of persons with the syndrome, half of whom develop pulmonary emboli. Swelling in the right calf with redness and tenderness, having respiratory symptoms like shortness of breath and chest pain, and having a cough that produced some bloody sputum are signs of DVT and pulmonary emboli. Headache is not conclusive for a clot in the cerebral vessels. Having trouble urinating and pain in the lower abdomen and groin are classic signs of a kidney stone.

A client presents to a health clinic complaining of several vague symptoms. As the history/physical continues, the health care provider clearly thinks the client may have myasthenia gravis. Which statements by the client would correlate with this diagnosis? Select all that apply. A) "Sometimes I have double vision." B) "I have more energy in the morning but get worse as the day goes by." C) "Sometimes I have numbness/tingling on my face." D) "I had what felt like an electric shock race down the back of my leg when I bend my neck." E) "I feel like I don't have enough energy to chew my food sometimes."

Ans: A,B,E A) "Sometimes I have double vision." B) "I have more energy in the morning but get worse as the day goes by." E) "I feel like I don't have enough energy to chew my food sometimes." Feedback: Now recognized as an autoimmune disease, myasthenia gravis is caused by an antibody-mediated destruction of acetylcholine receptors in the neuromuscular junction. This results in both muscle weakness and fatigability with sustained effort. Most commonly affected are the eye and periorbital muscles, with ptosis (drooping of eyelids) or diplopia (double vision) due to weakness of the extraocular muscles as an initial symptom. The disease may progress from ocular muscle weakness to generalized weakness. Chewing and swallowing may be difficult. In most persons, symptoms are least evident when arising in the morning, but grow worse with effort and as the day proceeds. Multiple sclerosis clients have paresthesias exhibited as numbness, tingling, burning sensations, or pressure on the face or involved extremities. The Lhermitte sign is an electric shock-like tingling down the back and onto the legs that is produced by flexion of the neck.

Which of the following clients are at high risk for developing anemia? Select all that apply. A) A HIV-positive client who is experiencing frequent infection and elevated CD4 counts B) A breast cancer client undergoing chemotherapy and radiation therapy C) A client who had a colectomy 3 weeks ago to remove adhesions and fecal impaction D) A COPD client with acute exacerbation requiring O2 therapy via C-PAP E) A middle-aged renal failure client who has hemodialysis three times/week

Ans: A,B,E A) A HIV-positive client who is experiencing frequent infection and elevated CD4 counts B) A breast cancer client undergoing chemotherapy and radiation therapy E) A middle-aged renal failure client who has hemodialysis three times/week Feedback: Anemia often occurs as a complication of infections, inflammation, and cancer. The most common causes of chronic disease anemias are acute and chronic infections, including AIDS and osteomyelitis; cancers; autoimmune disorders such as rheumatoid arthritis, systemic lupus erythematosus, and inflammatory bowel disease; and chronic kidney disease. Postsurgical clients do experience some blood loss and may become anemic but not always. COPD clients do not routinely have low hemoglobin levels.

The clinic nurse suspects the client is having a genetically determined hypersensitivity to common environmental allergens since the client is experiencing which of the following clinical manifestations? Select all that apply. A) Hives B) Runny nose C) Diarrhea D) Topical pustules E) Wheezes

Ans: A,B,E A) Hives B) Runny nose E) Wheezes Feedback: Localized, inherited allergic reactions mediated by IgE are known as atopic reactions, such as urticaria (hives). Atopic reactions are immediate (not delayed such as type IV poison ivy rash). Infections are not part of type I hypersensitivity. Autoimmune reactions are in response to antigens, not allergens.

Which assessment findings of a client with an elevated temperature would be considered a "normal" finding? Select all that apply. A) Flushed skin B) White, cheesy patches on oral mucous membranes C) Pain when moving joints to sit up in bed D) Unusual fatigue and drowsiness E) Complaining of "spots before the eyes"

Ans: A,C,D A) Flushed skin C) Pain when moving joints to sit up in bed D) Unusual fatigue and drowsiness Feedback: Common manifestations of fever are anorexia, myalgia, arthralgia, and fatigue. These discomforts are worse when the temperature rises rapidly or exceeds 39.5°C (103.1°F). Respiration is increased, and the heart rate usually is elevated. Dehydration occurs because of sweating and the increased vapor losses caused by the rapid respiratory rate. White, cheesy patches in the mouth are usually associated with a fungal infection. Spots before one's eyes are unrelated to fever.

Dysplasia

Dysplasia: The alteration in size, shape, and organization of adult cell types within a tissue.

An elderly client asks her health care provider if the reason she has developed aortic stenosis is because she drank so much milk as a child growing up on a farm. Which of the following responses is most accurate? A) "Drinking lots of real milk as a child may have contributed to the damage in your valve." B) "Atherosclerosis is a long process that eventually results in calcification of heart valves." C) "This calcification of your aortic valve is more than likely due to an undiagnosed thyroid problem." D) "More than likely, calcium has left your bones and collected on your aortic valve."

Ans: B B) "Atherosclerosis is a long process that eventually results in calcification of heart valves." Feedback: Dystrophic calcification represents the macroscopic deposition of calcium salts in injured tissue. Dystrophic calcification is commonly seen in atheromatous lesions of advanced atherosclerosis, in areas of injury in the aorta and large blood vessels, and in damaged heart valves. For example, calcification of the aortic valve is a frequent cause of aortic stenosis in the elderly. Drinking milk, undiagnosed thyroid problems, and calcium loss from bone causing osteoporosis do not cause damaged heart valves.

A college student has been called into the student health office because she tested positive for HIV on the enzyme-linked immunosorbent assay (ELISA). The student asks, "What is this Western blot assay going to tell you?" The best response by the health care provider is: A) "We always want two positive test results before we give you medicine." B) "The Western blot is a more sensitive assay that looks for the presence of antibodies to specific viral antigens." C) "This assay will actually look at all the individual cells in your blood and count how many HIV cells you have, so we can treat you with the proper medication." D) "If you are afraid of another blood test, we can do a rapid oral test to see if we get the same results."

Ans: B B) "The Western blot is a more sensitive assay that looks for the presence of antibodies to specific viral antigens." Feedback: The Western blot is a more sensitive assay than the EIA that looks for the presence of antibodies to specific viral antigens. In the case of a false-positive EIA result, the Western blot test can identify the person as uninfected. Technologic advances have led to new forms of testing, such as the oral test, home testing kits, and the new rapid blood test. Oral fluids contain antibodies to HIV. In the late 1990s, the FDA approved the OraSure test. The OraSure uses a cotton swab, which is inserted into the mouth for 2 minutes, placed in a transport container with preservative, and then sent to a laboratory for EIA and Western blot testing.

A client was diagnosed as HIV positive several years ago. Which of the following blood test results reflects the fact that the client now has developed AIDS? A) 350,000 platelets/μL B) CD4+ T-cell count less than 200 cells/μL C) Viral load 500,000 copies/mL D) White blood cell count of 5300 cmm

Ans: B B) CD4+ T-cell count less than 200 cells/μL Feedback:CD4+ cell counts are the primary measure that is used for the staging of HIV infection. Viral load, platelets, and leukocytes are also important assessments, but these are not used to stage HIV.

In the days following a tooth cleaning and root canal, a client has developed an infection of the thin, three-layered membrane that lines the heart and covers the valves. What is this client's most likely diagnosis? A) Pericarditis B) Endocarditis C) Myocarditis D) Vasculitis

Ans: B Endocarditis Feedback: The endocardium is a thin, three-layered membrane that lines the heart and covers the valves; infection of this part of the heart is consequently referred to as endocarditis.

A client with heart failure asks, "Why am I taking a 'water pill' when it's my heart that is having a problem?" While educating the client about the Frank-Starling mechanism, which of the following explanations is most appropriate to share? A) "You must be drinking way too many liquids. Your kidneys cannot filter all that you are drinking during the day." B) "Since your heart is not pumping efficiently, the kidneys are getting less blood flow; therefore, the kidneys are holding on to sodium and water." C) "Your heart muscle is overstretched, so it's not able to pump all the blood out. The prescribed 'water pills' help by decreasing your weight." D) "Since your heart function is impaired, the lungs are not able to oxygenate the blood and your kidneys are wearing out."

Ans: B) "Since your heart is not pumping efficiently, the kidneys are getting less blood flow; therefore, the kidneys are holding on to sodium and water." Feedback: In heart failure with a reduced ejection fraction, a decrease in cardiac output and renal blood flow leads to increased sodium and water retention by the kidney with a resultant increase in vascular volume and venous return to the heart and an increase in ventricular end-diastolic volume. Drinking water may increase volume but is not the physiological reason for retention of fluid. Diuretics do decrease weight as a result of diuresis, but weight loss is not the purpose for giving diuretics. The lungs are not the primary cause of heart failure.

A client asks, "Why do I have clogged arteries but my neighbor has higher 'bad cholesterol' levels and yet he is just fine?" The health care provider bases the reply on which of the following physiological principles about lipoprotein? A) "Your neighbor probably has higher amounts of good cholesterol (HDL) as well." B) "You more than likely have small, dense type of 'bad cholesterol' (LDL)." C) "Your neighbor has larger 'bad cholesterol' particles that can move into blood vessels but park in joints/tendons." D) "You must have a genetic predisposition to having clogged arteries."

Ans: B) "You more than likely have small, dense type of 'bad cholesterol' (LDL)." Feedback:There are different types of LDL, and some people with markedly elevated LDL do not develop atherosclerotic vascular disease, whereas other people with only modest elevations in LDL develop severe disease. Small, dense LDL is more toxic or atherogenic to the endothelium than large, buoyant LDL. It is more likely to enter the vessel wall, become oxidized, and trigger the atherosclerotic process.

A client with a history of heart failure has been referred for an echocardiogram. Results of this diagnostic test reveal the following findings: heart rate 80 beats/minute; end-diastolic volume 120 mL; and end-systolic volume 60 mL. What is this client's ejection fraction? A) 200 mL B) 50% C) 0.80 D) 180 mL

Ans: B) 50% Feedback:Ejection fraction = stroke volume ÷ end-diastolic volume, whereas stroke volume equals the difference between end-diastolic and end-systolic volume. Therefore, EF = 60 ÷ 120, or 50%.

A group of teenagers spent an entire day on the beach without using sunscreen. The first nght, their skin was reddened and painful to touch. The second day, they awoke to find large fluid-filled blisters over several body areas. The nurse recognizes the development of blisters as which type of inflammatory response? A) Cellular response B) Immediate transient response C) Continous response D) Delayed response

Ans: D) Delayed response

A client who has undergone a liver transplant 7 weeks ago has developed the following assessment data: ALT/AST elevation; jaundice of skin and sclera; weight gain with increase in abdominal circumference; and low-grade fever. The nurse suspects: A) Hyperacute graft rejection B) Acute graft rejection C) Chronic rejection D) Atherosclerosis of arteries of the liver

Ans: B) Acute graft rejection Feedback:Allograph rejection is caused by tissue incompatibility that causes the recipient immune cells to attack and destroy the donor cells. The opposite, graft versus host disease, is characterized by recipient immune cells being destroyed by the donor cells because the recipient cells are recognized by the donor cells as foreign.

Injured cells become very swollen as a result of: A) Increased cell protein synthesis B) Altered cell volume regulation C) Passive entry of potassium into the cell D) Bleb formation in the plasma membrane

Ans: B) Altered cell volume regulation

Atherosclerotic peripheral vascular disease is symptomatic with at least 50% occlusion. The primary peripheral symptom, due to ischemia, is: A) Edema B) Calf pain C) Varicosities D) Strong pulse

Ans: B) Calf pain

A client with diabetes has impiared sensation, circulation, and oxygenation of his feet. He steps on a piece of glass, the wound does not heal, and the tissue area becomes necrotic. The necrotic cell death is characterized by: A) Rapid apoptosis B) Cellular breakage C) Shrinkage and collapse D) Chronic inflammation

Ans: B) Cellular breakage

A 70-year-old male client has been admitted to a hospital for the treatment of a recent hemorrhagic stroke that has left him with numerous motor and sensory deficits. These deficits are most likely the rsult of which of the following mechanisms of cell injury? A) Free radical injury B) Hypoxia and ATP depletion C) Interference with DNA synthesis D) Impaired calcium homeostasis

Ans: B) Hypoxia and ATP depletion

Atherosclerotic plaque is most likely to be unstable and vulnerable to rupture when the plaque has a thin fibrous cap over a? A) Red thrombus B) Large lipid core C) Calcified lesion D) Vessel wall injury

Ans: B) Large lipid core

Pleuritic chest pain associated with respiratory movements is usually described as: A) Bilateral B) Localized C) Continuous D) Substernal

Ans: B) Localized Feedback:Pleuritis is usually unilateral and tends to be localized to the lower and lateral part of the chest; pain worsens with chest movements, such as deep breathing and coughing that accentuate pressure changes in the pleural cavity and increase movement of the inflamed or injured pleural surfaces. Musculoskeletal pain usually is bilateral and may occur as the result of frequent, forceful coughing. The pain associated with irritation of the bronchi usually is substernal and dull. Myocardial pain usually is located in the substernal area and is not affected by respiratory movements.

A client asks why his temperature is always below 98.6°F. The nurse responds: A) Some people maintain a core body temperature of 41°C and that is normal for them. B) Normal core temperature varies between individuals within the range of 97.0°F to 99.5°F. C) A person's highest point of core temperature is usually first thing in the morning. D) The best way to bring your body temperature up to normal is to live in a warmer climate.

Ans: B) Normal core temperature varies between individuals within the range of 97.0°F to 99.5°F. Feedback: Core temperature is normally maintained within a range of 36.0°C to 37.5°C (97.0°F to 99.5°F). A core temperature greater than 41°C (105.8°F) or less than 34°C (93.2°F) usually indicates that the body's thermoregulatory ability is impaired. Body heat is generated in the tissues of the body, transferred to the skin surface by the blood, and then released into the environment surrounding the body. The thermoregulatory center regulates the temperature of the deep body tissues, or "core" of the body, rather than the surface temperature. Internal core temperatures reach their highest point in late afternoon and evening and their lowest point in the early morning hours.

While sponging a client who has a high temperature, the nurse observes the client begins to shiver. At this point, the priority nursing intervention would be to: A) Administer an extra dose of aspirin B) Stop sponging the client and retake a set of vital signs C) Increase the room temperature by turning off the air conditioner and continue sponging the client with warmer water D) Place a heated electric blanket on the client's bed

Ans: B) Stop sponging the client and retake a set of vital signs Feedback: Modification of the environment ensures that the environmental temperature facilitates heat transfer away from the body. Sponge baths with cool water or an alcohol solution can be used to increase evaporative heat losses. More profound cooling can be accomplished through the use of a cooling blanket or mattress, which facilitates the conduction of heat from the body into the coolant solution that circulates through the mattress. Care must be taken so that cooling methods do not produce vasoconstriction and shivering that decrease heat loss and increase heat production.

A man sustained a puncture injury to his chest that caused a tension pneumothorax to form. This is a life-threatening condition because: A) Expired air exits the bleeding wound. B) Trapped, inspired air collapses the lung. C) The opposite lung hyperinflates. D) Blebs on the lung surface rupture.

Ans: B) Trapped, inspired air collapses the lung. Feedback: Tension pneumothorax occurs when the intrapleural pressure exceeds atmospheric pressure. It is a life-threatening condition and occurs when injury to the chest or respiratory structures permits air to enter but not leave the pleural space. Spontaneous pneumothorax occurs when an air-filled bleb, or blister, on the lung surface ruptures. Rupture of these blebs allows atmospheric air from the airways to enter the pleural cavity. This results in a rapid increase in pressure in the chest with a compression atelectasis of the unaffected lung.

The nurse caring for a population of HIV clients needs to be assessing for which of the following diseases that amounts to the leading cause of death for people with HIV? A) Leukemia B) Tuberculosis C) Pneumonia D) Toxoplasmosis

Ans: B) Tuberculosis Feedback:Although HIV-positive people can develop all types of cancer, have viral or bacterial pneumonia, and be infected with the parasite that causes toxoplasmosis, tuberculosis is the disease that leads to death most often. Tuberculosis (TB) is the leading cause of death for people with HIV infection worldwide and is often the first manifestation of HIV infection. In 2011, 23% of those with TB tested positive for HIV.

A client cut herself with a sharp knife while cooking dinner. The client describes how the wound started bleeding and had a red appearance almost immediately. The nurse knows that in the vascular stage of acute inflammation, the vessels: A) Bleed profusely until the body can compensate and start to send fibrinogen to the wound B) Vasodilate causing the area to become congested casuing the red color and warmth C) Contrict as a result of "fight/flight" hormone release resulting in pale-colored skin D) Swell to the point of compromising circulation causing the limb to become cool to touch

Ans: B) Vasodilate causing the area to become congested casuing the red color and warmth

Which of the following clients would be considered at high risk for developing pneumonia (both community and hospital setting)? Select all that apply. A) A teenager who spends a lot of time at local coffee shops using Wi-Fi to chat with friends B) A young adult in motorcycle accident with head injury requiring tracheostomy and mechanical ventilation C) A college female who is pregnant (unplanned) who has been consuming alcohol prior to positive pregnancy test D) A HIV-positive client with a WBC count of 2000 who has been camping near a commercial farm raising chickens for food E) A school-aged child with severe asthma controlled by steroids admitted for an exacerbation

Ans: B,D,E B) A young adult in motorcycle accident with head injury requiring tracheostomy and mechanical ventilation D) A HIV-positive client with a WBC count of 2000 who has been camping near a commercial farm raising chickens for food E) A school-aged child with severe asthma controlled by steroids admitted for an exacerbation Feedback: Persons requiring intubation and mechanical ventilation are particularly at risk, as are those with compromised immune function, chronic lung disease (like asthma), and airway instrumentation, such as endotracheal intubation or tracheotomy. Ventilator-associated pneumonia is pneumonia that develops in mechanically ventilated clients more than 48 hours after intubation. Neutropenia and impaired granulocyte function predispose to infections caused by S. aureus, Aspergillus, gram-negative bacilli, and Candida. Pneumonia in immunocompromised persons remains a major source of morbidity and mortality. The epithelial cells of critically and chronically ill persons are more receptive to binding microorganisms that cause pneumonia.

A family brings their elderly father to emergency department. He has been exposed to pneumococcal pneumonia at his retirement home. Today, they noted a change in his mental status. They thought he might need some oxygen. Which of the other assessment findings would correlate with this diagnosis? Select all that apply. A) Expiratory wheezes throughout all lung fields B) Increase in chest pain with deep inspiration C) Absent breathe sounds on the entire right side of the lung D) Loss of appetite for past few days E) Purulent sputum with bloody patches

Ans: B,D,E B) Increase in chest pain with deep inspiration D) Loss of appetite for past few days E) Purulent sputum with bloody patches Feedback:During the initial stage, coughing brings up watery sputum and breath sounds are limited, with fine crackles. As the disease progresses, the character of the sputum changes; it may be blood tinged or rust colored to purulent. Pleuritic pain, a sharp pain that is more severe with respiratory movements, is common. Elderly persons are less likely to experience marked elevations in temperature; in these persons, the only sign of pneumonia may be a loss of appetite and deterioration in mental status. Expiratory wheezes are usually associated with asthma. Absent breath sounds on an entire lung field usually is associated with pneumothorax or respiratory failure.

A motor vehicle accident has resulted in critical injury for the driver of the car. The driver has hit the steering wheel with his chest and fractured his sternum and some ribs. Which of the following manifestations would lead the staff to suspect the driver has developed a tension pneumothorax? Select all that apply. A) Audible friction rub over the affected lung B) Mediastinal shift of the trachea toward one side C) Marked peripheral edema in lower limbs and ascites D) Atrial fibrillation noted on ECG printout E) Subcutaneous emphysema palpated in the upper chest/neck region

Ans: B,E B) Mediastinal shift of the trachea toward one side E) Subcutaneous emphysema palpated in the upper chest/neck region Feedback:With tension pneumothorax, the structures in the mediastinal space shift toward the opposite side of the chest. There may be distention of the neck veins, subcutaneous emphysema (i.e., presence of air in the subcutaneous tissues of the chest and neck), and clinical signs of shock. Pneumothoraces do not typically cause a friction rub or dysrhythmias. Marked peripheral edema is associated with right-sided heart failure.

A 52-year-old man who is moderately obese has recently been diagnosed with hypertension by his primary care provider. Which of the client's following statements indicates a need for further health promotion teaching? A) "I've starting going to the gym before work three times a week." B) "I'm trying to cut back on the amount of salt that I cook with and add to my food." C) "I'm resolving to eat organic foods from now on and to drink a lot more water." D) "I'm planning to lose 15 pounds before the end of this year."

Ans: C) "I'm resolving to eat organic foods from now on and to drink a lot more water." Feedback: Weight loss, exercise, and salt reduction are all useful strategies in the management of hypertension. An organic diet and increased fluid intake are not known to reduce blood pressure.

Which of the following clients is at risk for developing a preventable disorder related to prolonged immobility? A) A middle-aged adult male diagnosed with bronchitis related to chronic smoking B) A young adult female diagnosed with sarcoidosis requiring corticosteroids to return her to remission C) A postsurgical client who is refusing to get out of bed and walk and will not wear those "uncomfortable elastic stocking" D) A sleep apnea client related to a history of smoking who utilizes a C-PAP machine every night at bedtime to maintain airway

Ans: C) A postsurgical client who is refusing to get out of bed and walk and will not wear those "uncomfortable elastic stocking" Feedback: A lack of mobility can result in secondary atelectasis (through incomplete lung expansion) and pulmonary embolism (from deep vein thrombosis). This is not the case with the other listed disorders of ventilation and gas exchange.

Because of its location, the presence of an abdominal aortic aneurysm may first be manifested as: A) Constipation B) Indigestion C) A pulsating mass D) Midabdominal pain

Ans: C) A pulsating mass Feedback: An abdominal aortic aneurysm may be noticed as a pulsating mass when the client is lying flat, once the aneurysm is rather large already. Although there may be pressure on the abdominal organs by the mass, it is not associated with constipation or indigestion. Severe midabdominal pain is a late sign of impending rupture.

Which of the following clients should most likely be assessed for orthostatic hypotension? A) A 78-year-old woman who has begun complaining of frequent headaches unrelieved by over-the-counter analgesics B) A 65-year-old client whose vision has become much less acute in recent months and who has noticed swelling in her ankles C) An 80-year-old elderly client who has experienced two falls since admission while attempting to ambulate to the bathroom D) A 42-year-old client who has a history of poorly controlled type 1 diabetes

Ans: C) An 80-year-old elderly client who has experienced two falls since admission while attempting to ambulate to the bathroom Feedback:Dizziness and syncope are characteristic signs and symptoms of orthostatic hypotension, and both predispose an individual to falls; this is especially the case among older adults. Headaches, edema, diabetes, and vision changes are not associated with orthostatic hypotension.

Glomerulonephritis is usually caused by: A) Vesicoureteral reflux B) Catheter-induced infection C) Antigen-antibody complexes D) Glomerular membrane viruses

Ans: C) Antigen-antibody complexes Feedback:Two types of immune mechanisms have been implicated in the development of glomerular disease: injury resulting from antibodies reacting with fixed glomerular antigens and injury resulting from circulating antigen-antibody complexes that become trapped in the glomerular membrane. Reflux, which is the most common cause of chronic pyelonephritis, results from superimposition of infection on congenital vesicoureteral reflux or intrarenal reflux. Urinary catheters provide a means for microorganisms to ascend into the urinary tract to cause bladder infections or pyelonephritis.

A client is diagnosed with an abdominal aortic aneurysm that the physician just wants to "watch" for now. When teaching the client about signs/symptoms to watch for, the nurse will base the teaching on which of the following physiological principles? A) Small diameter of this vessel will cause it to rupture more readily. B) The larger the aneurysm, the less tension placed on the vessel. C) As the aneurysm grows, more tension is placed on the vessel wall, which increases the risk for rupture. D) The primary cause for rupture relates to increase in abdominal pressure such as straining to have a bowel movement.

Ans: C) As the aneurysm grows, more tension is placed on the vessel wall, which increases the risk for rupture. Feedback: Because the pressure is equal throughout, the tension in the part of the balloon with the smaller radius is less than the tension in the section with the larger radius. The same holds true for an arterial aneurysm in which the tension and risk of rupture increase as the aneurysm grows in size. Wall tension is inversely related to wall thickness, such that the thicker the vessel wall, the lower the tension, and vice versa. Although arteries have a thicker muscular wall than veins, their distensibility allows them to store some of the blood that is ejected from the heart during systole, providing for continuous flow through the capillaries as the heart relaxes during diastole.

A client presents to the clinic with a swollen, painful "hang nail" on the index finger. There is a large pustule over the site that needs to be lanced. The health care worker knows that which mediator of inflammation causes this increase in capillary permeability and pain? A) Serotonin B) Histamine C) Bradykinin D) Nitric oxide

Ans: C) Bradykinin Feedback: Bradykinin causes increased capillary permeability and pain. Serotonin and histamine are released by the mast cell degranulation. Histamine causes arteriole dilation and increased permeability of venules. Serotonin has actions similar to those of histamine. Nitric oxide relaxes smooth muscle and reduces platelet aggregation and adhesion.

A client underwent an open cholecystectomy 4 days ago, and her incision is now in the proliferative phase of healing. What is the dominant cellular process that characterizes this phase of the client's healing? A) Hemostasis and vasoconstriction B) Keloid formation C) Collagen secretion by fibroblasts D) Phagocytosis by neutrophils

Ans: C) Collagen secretion by fibroblasts

The client is found to have liver disease, resulting in the removal of a lobe of his liver. Adaptation to be reduced size of the liver leads to which phenomenon in the remaining liver cells? A) Metaplasia B) Organ atrophy C) Compensatory hyperplasia D) Physiologic hypertrophy

Ans: C) Compensatory hyperplasia

Contact with poison ivy has resulted in intense pruritus, erythema, and weeping on a client's forearm. Which of the following processes resulted in the client's signs and symptoms? A) IgE-mediated mast cell degranulation B) Formation of antigen-antibody complexes C) Cytokine release by sensitized T cells D) Formation of antibodies against cell surface antigens

Ans: C) Cytokine release by sensitized T cells Feedback:The characteristic reaction to poison ivy is an example of contact dermatitis, a type IV hypersensitivity reaction. As such, the reaction is caused by sensitized T cells and the release of cytokines. IgE-mediated mast cell degranulation causes type I reactions, whereas antigen-antibody complexes are associated with type III reactions. Antibody formation against cell surface antigens are associated with type II reactions.

A client has experienced significant decreases in mobility and stamina during a 3-week hospital stay for the treatment of femoral head fracture. Which of the following phenomena most likely accounts for the client's decrease in muscle function? A) Impiared muscle cell metabolism resulting from metaplasia B) Dysplasia as a consequence of inflammation during bone remodeling C) Disuse atrophy of muscle cells during a prolonged period of immobility D) Ischemic atrophy resulting from vascular changes while on bedrest

Ans: C) Disuse atrophy of muscle cells during a prolonged period of immobility

A mother rushes her toddler into the emergency department stating, "My baby can't breathe." Initial assessment reveals the child is struggling to breathe in an upright position. He has both inspiratory and expiratory stridor and is using his chest muscles to breath. The nurse suspects the child has which of the following acute respiratory infections? A) Croup B) Asthma C) Epiglottitis D) Bronchiolitis

Ans: C) Epiglottitis Feedback: Epiglottitis typically presents with an acute onset of sore throat and fever. The child appears pale, toxic, and lethargic and assumes a distinctive position—sitting up with the mouth open and the chin thrust forward. Symptoms rapidly progress to difficulty swallowing, a muffled voice, drooling, and extreme anxiety. Moderate to severe respiratory distress is evident. There are inspiratory and sometimes expiratory stridor, flaring of the nares, and inspiratory muscle retractions. Croup or acute laryngotracheobronchitis is a viral infection that affects the larynx, trachea, and bronchi. Acute bronchiolitis is a viral infection of the lower airways, most commonly caused by the respiratory syncytial virus. Asthma is a reactive airway disease rather than an infection.

A 9-month-old infant has been diagnosed with botulism after he was fed honey. The child's mother was prompted to seek care because of this child's sudden onset of neuromuscular deficits, which were later attributed to the release of substances by Clostridium botulinum. Which virulence factor contributed to this child's illness? A) Endotoxins B) Adhesion factors C) Exotoxins D) Evasive factors

Ans: C) Exotoxins Feedback:Exotoxins are proteins released from the bacterial cell during growth, as in the case of botulism poisoning. Adhesion factors, evasive factors, and endotoxins are not evident in this release of botulinum toxin.

A client with diabetes and severe peripheral vascular disease has developed signs of dry gangrene on the great toe of one foot. The client asks, "How this can happen?" Which of the following pathphysiologic processes should the nurse explain to this client? "More than likely, your gangrene is caused by: A) Inappropriate activation of apoptosis, which means death of your cells." B) Bacterial invasion into the foot and toe." C) Impaired arterial blood supply to your toe." D) Metaplastic cellular changes in your toe."

Ans: C) Impaired arterial blood supply to your toe."

During lecture on inflammation, the physiology instructor discusses the major cellular components involved in the inflammation response. The instructor asks, "Which of the following cells arrives early in great numbers?" The student with the correct response is: A) Basophils B) Lymphocites C) Neutrophils D) Monocytes

Ans: C) Neutrophils

A diabetic client has injured his foot while walking barefoot on the lawn. On admission, which of the following assessment findings would be considered a localized cardinal sign of acute inflammation? A) Temperature of 101 F B) Fatigue and listlessness C) Redness and edema at injured site D) Urine output of less than 500mL/24 hours (low)

Ans: C) Redness and edema at injured site

Following a kitchen accident with a knife, the client's cut has experienced a decrease in the amount of bleeding and has developed a clot. The nurse knows this is primarily a result of humoral control of blood flow with the release of: A) Histamine B) Bradykinin C) Serotonin D) Prostaglandin E2

Ans: C) Serotonin Feedback: Serotonin release causes vasoconstriction of blood vessels and plays a major role in control of bleeding. Histamine, bradykinin, and prostaglandin E2 cause vasodilation of blood vessels.

A client arrived in the emergency department following 2 days of general malaise. The temperature is 103.8°F. The nurse administers the prescribed aspirin, an antipyretic. Which of the following statements relates to the rationale for this action? A) Temperatures in excess of 37.5°C (99.5° F) can result in seizure activity. B) Lower temperatures inhibit the protein synthesis of bacteria. C) These medications protect vulnerable organs, such as the brain, from extreme temperature elevation. D) Most common antipyretics have been shown to have little effect on core temperature.

Ans: C) These medications protect vulnerable organs, such as the brain, from extreme temperature elevation. Feedback: There is little research to support the belief that fever is harmful unless the temperature rises to extreme levels; it has been shown that small elevations in temperature, such as those that occur with fever, enhance immune function. Antipyretics are effective in lowering core body temperature.

A client has been diagnosed with gram-negative pneuomonia of the lower lobe. Knowing that gram-negative bateria have a lipopolysaccharide endotoxin on their outer cell membrane, the health care provider should be assessing the client for which pathophysiological end result? A) Damage to cellular mitochondria B) Increased ATP levels C) Activation of the p53 protein D) Apoptosis

Ans: D) Apoptosis

A client has been admitted to the hospital for the treatment of HIV infection, which has recently progressed to overt AIDS. Which of the following nursing actions should the nurse prioritize when providing care for this client? A) Frequent neurologic vital signs and thorough skin care B) Hemodynamic monitoring and physical therapy C) Careful monitoring of fluid balance and neurologic status D) Astute infection control and respiratory assessments

Ans: D) Astute infection control and respiratory assessments Feedback:Although all of the cited assessments and interventions may be of some value, infection control and the early identification of potential respiratory infections are paramount in the care of clients with AIDS.

Following several weeks of increasing fatigue and a subsequent diagnostic workup, a client has been diagnosed with mitral valve regurgitation. Failure of this heart valve would have which of the following consequences? A) Backup of blood from the right atrium into the superior vena cava B) Backflow from the right ventricle to the right atrium during systole C) Inhibition of the SA node's normal action potential D) Backflow from the left ventricle to the left atrium

Ans: D) Backflow from the left ventricle to the left atrium Feedback: The mitral valve separates the left ventricle from the left atrium; failure of this valve would cause backflow from the former to the latter during systole. Valve function does not directly affect cardiac contractility.

Which elevated serum marker for systemic inflammation is now considered a major risk factor for atherosclerosis and vascular disease? A) Leukocytosis B) Homocysteine C) Serum lipoprotein D) C-reactive protein

Ans: D) C-reactive protein Feedback: CRP is an acute-phase reactant synthesized in the liver that is a marker for systemic inflammation. A number of population-based studies have demonstrated that baseline CRP levels can predict future cardiovascular events among apparently healthy individuals. High-sensitivity CRP (hs-CRP) may be a better predictor of cardiovascular risk than lipid measurement alone. Homocysteine and serum lipoprotein are also serum markers, but they do not identify inflammation. Leukocytosis is an indicator of infection rather than inflammation alone.

The client is immobilized following a hip injury and has begun demonstrating lower leg discoloration with edema, pain, tenderness, and increased warmth in the midcalf area. He has many of the manifestations of: A) Stasis ulcerations B) Arterial insufficiency C) Primary varicose veins D) Deep vein thrombosis

Ans: D) Deep vein thrombosis

During a client's admission assessment prior to reduction mammoplasty surgery, the nurse notes a reference to a Leiden mutation in the client's history. The nurse assesses this client for an increased risk of developing which of the following postsurgical complications? A) Hemorrhage B) Myocardial infarction C) Hemophilia A or B D) Deep vein thrombosis

Ans: D) Deep vein thrombosis Feedback: The Leiden mutation predisposes to venous thrombosis. It is not associated with hemophilia or an increased risk of hemorrhage. The client's risk of deep vein thrombosis likely supersedes her risk of myocardial infarction.

In addition to direct invasion of the vascular wall by an infectious agent, this pathogenic mechanism is a common cause of vasculitis? A) Necrotizing granulomatous B) Tissue necrosis C) Mononuclear cells D) Immune-mediated inflammation

Ans: D) Immune-mediated inflammation Feedback:The two most common pathogenic mechanisms of vasculitis are direct invasion of the vascular wall by an infectious agent and immune-mediated inflammation. The most common mechanisms that initiate noninfectious vasculitis are pathological immune responses that result in endothelial activation, with subsequent vessel obstruction, and ischemia of the dependent tissue. In almost all forms of vasculitis, the triggering event initiating and driving the inflammatory process is unknown. Medium-size vessel vasculitides produce necrotic tissue damage. Large-vessel vasculitides involve mononuclear cells. Wegener granulomatosis is characterized by a triad of acute necrotizing granulomatous lesions of the upper respiratory tract (ear, nose, sinuses, and throat), necrotizing vasculitis of the affected small- to medium-sized vessels of the lungs and respiratory airways, and renal disease in the form of focal necrotizing glomerulonephritis.

A client has begun taking acyclovir, an antiviral medication, to control herpes simplex outbreaks. What is this drug's mechanism of action? A) Inhibition of viral adhesion to cells B) Elimination of exotoxin production C) Antagonism of somatic cell-binding sites D) Interference with viral replication processes

Ans: D) Interference with viral replication processes Feedback:Acyclovir, like most antiviral drugs, interferes with normal viral replication. Antiviral drugs do not normally affect adhesion, and viruses are not associated with the production of exotoxins.

During lecture on wound care, the instructor mentions the final stage of the cellular response of acute inflammation. Of the following statements, which describes what physiologically occurs in the final stage? A) Leukocytes accumulate and begin migration to the site of injury. B) Chemokines direct teh trafficking leukocytes C) Mediators are transformed into inactive metabolites D) Neutrophils, monocytes, and macrophages engulf and degrade the bacteria/cellular debris

Ans: D) Neutrophils, monocytes, and macrophages engulf and degrade the bacteria/cellular debris

A client with a long-standing diagnosis of human immunodeficiency virus (HIV) has recently developed neutropenia and been admitted to a hospital. Which of the following measures should be prioritized by the nurses who are providing his care? A) Administration of prophylactic antibiotics B) Supplementary oxygen and administration of bronchodilators C) Administration of antiretroviral medications D) Vigilant handwashing to protect against severe bacterial infections

Ans: D) Vigilant handwashing to protect against severe bacterial infections Feedback:Neutropenia carries a greatly increased risk of infection that necessitates vigilant infection control measures and strict adherence to standard precautions such as handwashing. Antiretrovirals do not directly address his neutropenia, and antibiotics are not normally provided prophylactically. The client is susceptible to respiratory infections, but therapeutic interventions are not required until indicated by a diagnosis such as pneumonia.

Venous thrombosis most commonly occurs in the lower extremities. Risk factors for venous thrombosis include which of the following? A. Stasis of blood, hypercoagulability, inflammation B. Hypocoagulability, vessel vall injury, increased pressure on deep veins C. Vessel wall injury, hypocoagulability, decreased venous blood flow D. Stasis of blood, hypercoagulability, vessel wall injury

Answer: D. Stasis of blood, hypercoagulability, vessel wall injury Rationale: In 1846, Virchow described the triad that has come to be associate with venous thrombosis: stasis of blood, increased blood coagulability, and vessel wall injury. Inflammation is a symptom of venous trhombosis, not a risk factor. Deceased venous blood flow can occur because of benous thrombosis, if the thrombus does not completely obstuct the bein; it is not a risk factor. Hypocoagulability would not cause a thrombus to form.

Signs and Symptoms of problems with this valve

Aortic valve regurgitation ● Most remain asymptomatic during this phase. The only sign may be a soft systolic aortic murmur. Mitral valve regurgitation ● Symptoms are those LV failure such as dyspnea on exertion, paroxysmal nocturnal dyspnea, and orthopnea.

Know the differences: apoptosis, cellular rupture, shrinkage, chronic inflammation

Apoptosis - Programmed cell death with no inflammatory response; normal occurrence in the body which helps remove damaged cells and regulate tissue regeneration; involves shrinkage of cellular structures, fragmentation and separation into apoptotic bodies, and engulfment by phagocytic cells. Cellular Rupture (or cellular breakage) - Often part of necrotic cell damage, cellular rupture occurs after swelling and leads to inflammation; generally unregulated Shrinkage - Hallmark step in apoptosis; can be due to presence of hypertonic solution; can be due to dry gangrene Chronic Inflammation - Self perpetuating inflammation that can last weeks, months or years; results from infiltration of macrophages, lymphocytes, plasma cells and proliferation of fibroblasts.

Disease progression—> Abdominal Aortic Aneurysm

As the aneurysm expands, it may compress the lumbar nerve roots, causing lower back pain that radiates to the posterior aspects of the legs. The aneurysm may extend to and impinge on the renal, iliac, or mes- enteric arteries, or to the vertebral arteries that supply the spinal cord. An abdominal aneurysm also may cause erosion of vertebrae. Stasis of blood favors thrombus formation along the wall of the vessel and peripheral emboli may develop, causing symptomatic arterial insufficiency. Measures to slow aneurysm growth and lower the risk of rupture include risk factor modification. Hypercholesterolemia and high blood pressure should be controlled and smoking discontinued. If untreated, aneurysm could lead to AORTIC DISSECTION, an acute, life threatening condition. It involves hemorrhage into the vessel wall with longitudinal tearing of the vessel wall to form a blood-filled channel. Aortic dissection is caused by conditions that weaken or cause degenerative changes in the elastic and smooth muscle layers of the aorta.

What is atelectasis and how do you prevent it?

Atelectasis- - Incomplete expansion of a lung or portion of lung - Caused by: - Airway obstruction, lung compression (pneumothorax/ pleural effusion), increased recoil of the lung dies to loss of pulmonary surfactant Etiology and Pathogenesis - Primary atelectasis → Of newborn (present at birth) - Lung has never been inflated - Seen frequently with premature and high-risk infants - Secondary form→ Develops in the neonatal period or later in life - Infants who established respiration and subsequently experienced impairment of lung expansion - Respiratory distress syndrome associated with lack of surfactant - Airway obstruction due to aspiration of amniotic fluid or blood - Bronchopulmonary dysplasia - Acquired atelectasis → Occurs mainly in adults - Caused most commonly → Airway obstruction and lung compression - Danger of obstructive atelectasis increase after surgery - Immobility, narcotics, pain, or anesthesia - Promote retention of viscid bronchial secretions causing obstruction - Encourage deep breathing, frequent change of position, adequate hydration - Compression of lung tissue leads to atelectasis → Another cause - Common in people with pleural effusion from CHF Prevention - Deep breathing exercises and coughing after surgery can reduce your risk of developing atelectasis. If you smoke, you can lower your risk of developing the condition by quitting smoking before any operation - Encourage your patient to do deep breathing techniques, incentive spirometer

What does atherosclerosis cause in the heart vessels?

Atherosclerosis: macrophages ingest LDL → oxidized LDL released → endothelium responds with inflammation and formation of plaque (scarring) in heart vessels Signs & symptoms (SxS): Generally no SxS → goes unnoticed/unchecked How effects heart: Plaque blocks arteries → decreases flexibility of structure ● Narrows & hardens heart vessels ● Reduces blood flow → reduces O2 flow in/out of heart Risks: ● Coronary Artery Disease (CAD) ● Myocardial Infarction (MI) ● Heart Failure (any cardiac event)

Atrophy

Atrophy: A wasting or diminution of size, often accompanied by a decrease in function of a cell, tissue, or organ. For example: middle aged female experiencing menopause due to LOSS of estrogen stimulation which results in atrophic changes in the reproductive system. Example 2: A person who has lost function in their legs will experience muscle atrophy due to their legs not being used.

Brain natriuretic peptide (BNP) (Patho Textbook 4th ed Chapter 20,page 490 &496)

BNP along with ANP belongs to a group of natriuretic peptides that are produced and secreted by the heart muscle.Natriuretic peptides have potent diuretic, natriuretic, vascular smooth muscle, and other neurohumoral actions that affect cardiovascular function. ● It is commonly associated with heart failure just like ANP. ● B-type natriuretic peptide (BNP), so named because it was originally found in extracts of the porcine brain, is primarily secreted by the ventricles as a response to increased ventricular pressure or fluid overload. ● Circulating levels of both ANP and BNP are elevated in persons with heart failure. The concentrations are well correlated with the extent of ventricular dysfunction, increasing up to 30-fold in persons with advanced heart disease ● BNP are used clinically in the diagnosis of heart failure, to evaluate the severity of left ventricular compromise, estimate the prognosis, and evaluate the effectiveness of treatment. ● BNP may not be as sensitive in persons with heart failure who are obese or have renal failure.

DNA Synthesis signs and symptoms

DNA Synthesis: - DNA modification and damage can occur as a result of oxidative stress. - Oxidative stress is essentially an imbalance between the production of free radicals and the ability of the body to counteract or detoxify their harmful effects through neutralization by antioxidants. - Oxidative stress can lead to oxidative of cell components, activation of signal transduction pathways, and changes in gene and protein expression. - ex) Mutation in gene for superoxide dismutaste are linked with amyotrophic lateral sclerosis (ALS)

Patient Education on ejection fraction

Ejection fraction refers to the amount of blood the heart pumps to all parts of the body. There is a percentage used here, but basically tells us if the heart is functioning properly to deliver blood to all the necessary areas in the body. If the percentage is lower than the normal value, it can tell us that there is something wrong with the heart and is a cause for concern. Prioritize telling them what ejection fraction is, talk about HTN leading to heart pump not working as well, say things that would lead to aggravation, like excessive alcohol, salt, not taking meds. Want to make sure it is on the appropriate education level, what they can do to have the best quality of life.

Know the difference between: Endocarditis, Pericarditis, Myocarditis

Endocarditis: inflammation of the inner (lining) of the heart (particularly heart valves) Pericarditis: inflammation of myocardium; most often caused by viral or bacterial infection Myocarditis: inflammation of the membrane surrounding the heart, the pericardium

Know the difference between: Epiglotitis, Asthma, Croup, Bronchiolitis (know etiology, signs and symptoms and pathogenesis)

Epiglottitis typically presents with an acute onset of sore throat and fever. The child appears pale, toxic, and lethargic and assumes a distinctive position—sitting up with the mouth open and the chin thrust forward. Symptoms rapidly progress to difficulty swallowing, a muffled voice, drooling, and extreme anxiety. Moderate to severe respiratory distress is evident. There are inspiratory and sometimes expiratory stridor, flaring of the nares, and inspiratory muscle retractions. Croup or acute laryngotracheobronchitis is a viral infection that affects the larynx, trachea, and bronchi. Acute bronchiolitis is a viral infection of the lower airways, most commonly caused by the respiratory syncytial virus. Asthma is a reactive airway disease rather than an infection.

Free radical injury signs and symptoms

Free radical injury: Important key players: Nitric Oxide (NO-) and Oxygen-derived free radicals. - NO-: smooth muscle relaxation, antagonism of platelet adhesion, - aggregation, degranulation, and leukocyte recruiter. It has antimicrobial actions and is a host mediator against infection. - Oxygen-derived free radicals- O2 (superoxide radical), and OH (hydroxyl radical). They both combine with NO- to form other reactive nitrogen intermediates which increases inflammatory process and cause more tissue injury. - Free radicals react with proteins, lipids, and carbohydrates and damage cell membranes, inactivate enzymes, and damage nucleic acid that make up DNA (which can lead to mutations → can cause cancer) - Free radicals destroy phospholipids in the cell membrane - Sign/Symptoms of a free radical injury- Swelling, formation of exudates to abscess formation or ulceration, cancer

Know what sign/symptom a patient may have if they have a free radical injury, hypoxia and ATP depletion, interference with DNA synthesis, impaired calcium homeostasis

Free radical injury: Important key players: Nitric Oxide (NO-) and Oxygen-derived free radicals. - NO-: smooth muscle relaxation, antagonism of platelet adhesion, - aggregation, degranulation, and leukocyte recruiter. It has antimicrobial actions and is a host mediator against infection. - Oxygen-derived free radicals- O2 (superoxide radical), and OH (hydroxyl radical). They both combine with NO- to form other reactive nitrogen intermediates which increases inflammatory process and cause more tissue injury. - Free radicals react with proteins, lipids, and carbohydrates and damage cell membranes, inactivate enzymes, and damage nucleic acid that make up DNA (which can lead to mutations → can cause cancer) - Free radicals destroy phospholipids in the cell membrane - Sign/Symptoms of a free radical injury- Swelling, formation of exudates to abscess formation or ulceration, cancer Hypoxia: - Hypoxia: Deprives the cell of oxygen and interrupts oxidative metabolism and the generation of ATP. - During hypoxic conditions, the body compensates by stimulating red blood cell formation and increasing the formation of new blood vessels. - Signs and symptoms of hypoxia- Increase heart rate, metabolic acidosis, peripheral vasoconstriction, diaphoresis, increase BP. - Impaired arterial blood flow = dry gangrene, wet gangrene = impaired venous gangrene ATP depletion: - Pathogenesis is related to hypoxia and even ischemia. - Important effect = acute cell swelling caused by failure of energy-dependent sodium/potassium membrane pump. Leading to potassium decrease and sodium and water retention in the cell. Where sodium goes water follows! DNA Synthesis: - DNA modification and damage can occur as a result of oxidative stress. - Oxidative stress is essentially an imbalance between the production of free radicals and the ability of the body to counteract or detoxify their harmful effects through neutralization by antioxidants. - Oxidative stress can lead to oxidative of cell components, activation of signal transduction pathways, and changes in gene and protein expression. - ex) Mutation in gene for superoxide dismutaste are linked with amyotrophic lateral sclerosis (ALS) Impaired Calcium Homeostasis: - Ischemia and other toxic injuries causes the impaired mitochondria to release stored calcium. - Calcium functions as an important second messenger and cytosolic signal for cell response - Calcium/calmodulin dependent kinases indirectly mediates the effects of calcium on smooth muscle contraction and - glycogen breakdown. - Calcium levels are supposed to be low and it is maintained by calcium/magnesium ATPase exchange system. - Increase calcium levels can damage cell membrane and cytoskeleton and deplete ATP

What is the reason for a patient to have gangrene?

Gangrene: mass of tissue undergoes necrosis due to either a lack of blood flow OR a serious bacterial infection Dry gangrene results from interference with arterial blood supply to a part without interference with venous return. It is a form of coagulative necrosis. It has a line of demarcation between dead tissue and healthy tissue. It is confined exclusively to extremities. - Characterized by dry and shriveled skin ranging in color from brown to purplish blue or black - Most common in people who have arterial blood vessel disease, such as atherosclerosis, or people who have diabetes. Moist gangrene results from interference with venous return from the part. The area is cold, swollen, and pulseless. There is no line of demarcation and the spread of tissue damage is rapid and it can affect internal organs or extremities. It is wet/moist due to bacterial infection. Gas gangrene results from infection of devitalized tissue by Clostridium perfringens. The area develops bubbles of hydrogen sulfide that form in the muscle and it may be fatal. (could result in amputation).

Know Heart Failure (Ch.20 4th ed. pg 486)

Heart failure: functional or structural dysfunction of the filling or emptying of the ventricles of the heart ● Know the lecture slides - left vs. right sided heart failure, know alteration and structure and how it affects function ● Causes: coronary artery disease (CAD), hypertension, dilated cardiomyopathy, valvular heart disease, age (geriatic population at higher risk)

Hypertensive Crisis

Hypertensive crisis refers to BP of greater than 180 mmHg systolic or diastolic pressure greater than 120 mmHg. Feels agitated, chin strap feeling, headache, restless, confused, pounding pulse in ears, blurry vision. ● Can be broken down into 2 classifications ○ Hypertensive urgency ■ Marked elevated BP, but without rapid progression of organ involvement ○ Hypertensive emergency ■ When marked elevated BP is related to signs, symptoms or lab evidence of end-organ damage, such as CHF or acute renal failure (they stop peeing). ■ Brain, heart, and/or kidney often invloved

Hypoxia signs and symptoms

Hypoxia: - Hypoxia: Deprives the cell of oxygen and interrupts oxidative metabolism and the generation of ATP. - During hypoxic conditions, the body compensates by stimulating red blood cell formation and increasing the formation of new blood vessels. - Signs and symptoms of hypoxia- Increase heart rate, metabolic acidosis, peripheral vasoconstriction, diaphoresis, increase BP. - Impaired arterial blood flow = dry gangrene, wet gangrene = impaired venous gangrene

Valve Failure Results?

If AV valves leading to ventricles don't work → mitral or tricuspid problems - mitral valve regurgitation: blood flows back into the left atrium during systole → rigid/thick valve → Inc in left ventricular end-diastole → inc in stroke volume → inc in left atria → enlarged, hypertrophied left ventricle, hyperdynamic left ventricular impulse, and pansystolic heart murmur and predisposes atrial fibrillation & pulmonary congestion If semilunar valves leading out of ventricles don't work → aortic or pulmonary problems - aortic valve regurgitation: blood flows back into the left ventricle during diastole 1. acute aortic regurgitation, the heart tries to compensate for a sudden, large regurgitant volume (which still fails to maintain cardiac output) → severe elevation in Left ventricular end diastolic pressure → pulmonary edema → dec in cardiac output → sympathetic activation → increase in Heart rate and peripheral vascular resistance → the regurgitation worsens → death from pulmonary edema, ventricular arrhythmias, or circulatory collapse 2. Chronic aortic regurgitation: a gradual onset, combined left ventricular volume and pressure overload - Valve deformity increases, increased regurgent flow into the left ventricle → diastolic BP falls → left ventricle enlarges and hypertrophies → stroke volume increases to maintain cardiac output until late in the course of the disease (so many people are asymptomatic for many decades) → signs of Left ventricular failure begin to appear: exertional dyspnea, orthopnea, and paroxysmal nocturnal dyspnea - Drop in diastolic BP→ decrease in coronary perfusion - Angina (rare) - Arterial pulse pressure increase, elevated systolic pressure and low diastolic pressure (due to large stroke volume) → prominent, throbbing carotid & peripheral pulses, left ventricular pulse that moves the chest with each beat - Severe cases: water - hammer pulse: distention and collapse of artery - aortic valve regurgitation can cause complications, including: heart failure, infections (ie endocarditis), heart rhythm abnormalities, death

Impaired Calcium Homeostasis signs and symptoms

Impaired Calcium Homeostasis: - Ischemia and other toxic injuries causes the impaired mitochondria to release stored calcium. - Calcium functions as an important second messenger and cytosolic signal for cell response - Calcium/calmodulin dependent kinases indirectly mediates the effects of calcium on smooth muscle contraction and - glycogen breakdown. - Calcium levels are supposed to be low and it is maintained by calcium/magnesium ATPase exchange system. - Increase calcium levels can damage cell membrane and cytoskeleton and deplete ATP

What causes ischemia and other toxic injuries that causes the accumulation of intracellular calcium? (chapter 2 4th edition)

Ischemia: Deficit of oxygen in the cells (i.e., decreased blood flow due to vasoconstriction or vascular obstruction) (can lead to hypoxia) What is Normal? Normally, intracellular calcium ion levels are kept extremely low compared with extracellular levels. This is maintained by energy-dependent calcium/magnesium-ATPase. Not Normal: Ischemia and certain toxins lead to an increase in cytosolic calcium because of the increased influx across the cell membrane and the release of calcium from intracellular stores (mitochondria and SER). The increased calcium may activate a number of enzymes with potentially damaging effects. Phospholipases can damage the cell membrane. Proteases can damage the cytoskeleton and membrane proteins. ATPases can break down ATP. Endonucleases can fragment chromatin.

Ischemic Atrophy

Ischemic Atrophy: Decreased blood supply to a body organ or part due to functional constriction or obstruction of a blood vessel leading to that part of the body organ to atrophy in size.

Lung compliance - what is it? Who does it affect?

Lung compliance refers to the ease with how the lungs can be inflated ie how easy the lungs can expand under pressure. For the lungs to function properly there must be adequate surfactant and mature lungs. This will allow alveoli to stay open between breaths. Lung compliance is determined by the 1. elastic properties of the lungs and 2. Alveolar surface tension Think of a balloon that has been previously inflated compared to a new ballon that is stiff anon compliant (which balloon will be easier to blow up?) Lung Compliance affect: Premature infants with RDS, COPD,Smokers, Cystic Fibrosis

Metaplasia

Metaplasia: Change in type of adult cell in a tissue to a form that is not normal for that tissue. (adult cell→ different adult cell) - Ex. Callus

Know the differences: metaplasia, dysplasia, atrophy, ischemic atrophy

Metaplasia: Change in type of adult cell in a tissue to a form that is not normal for that tissue. (adult cell→ different adult cell) - Ex. Callus Dysplasia: The alteration in size, shape, and organization of adult cell types within a tissue. Atrophy: A wasting or diminution of size, often accompanied by a decrease in function of a cell, tissue, or organ. For example: middle aged female experiencing menopause due to LOSS of estrogen stimulation which results in atrophic changes in the reproductive system. Example 2: A person who has lost function in their legs will experience muscle atrophy due to their legs not being used. Ischemic Atrophy: Decreased blood supply to a body organ or part due to functional constriction or obstruction of a blood vessel leading to that part of the body organ to atrophy in size.

Portals of entry (4th edition Chapter 14, pg. 307 or pg. 331 pdf)

Microbes can enter the host by direct contact, ingestion, and inhalation. The portal of entry does not dictate the site of infection. Ingested pathogens may penetrate the intestinal mucosa, disseminate through the circulatory system, and cause diseases in other organs such as the lung or liver. Whatever the mechanisms of entry, the transmission of infectious agents is directly related to the number of infectious agents absorbed by the host. ● Penetration ○ Any disruption in the integrity of the body's surface barrier - skin or mucous membranes - is a potential site for invasion of microorganisms. The break could be an abrasion, burn, or penetrating wound. ■ Accidental (skin barrier break) ● Stabbed, nail, cut ■ Planned ● Injection (if unclean, can become aseptic because introducing bacteria into blood) ■ Primary infectious processes ● CHickenpox or impetigo (rash) ● Direct Contact ○ Some pathogens are transmitted directly from infected tissue or secretions to exposed, intact mucous membranes (Ex. STI's) ■ Vertical transmission- mother to child via placenta or during birth ● Ingestion ○ The entry of pathogenic microorganisms or their toxins through the oral cavity and gastrointestinal tract represents one of the more efficient means of disease transmission in humans. ■ Many bacteria, viral, and parasitic infections are initiated through the ingestion of contaminated food and water ● Inhalation ○ The respiratory tract of a healthy person is equipped with a multilayered defense system to prevent potential pathogens from entering the lungs. ○ The surface of the respiratory tree is lined with a layer of mucus that is continuously swept up and away from the lungs and toward the mouth by the beating motion of ciliated epithelial cells. ○ Coughing aids in the removal of particulate matter from the lower respiratory tract. Respiratory secretions contain antibodies and enzymes capable of inactivating infectious agents and particulate matter that reaches the lungs are cleared by phagocytic cells. ○ DESPITE THE IMPRESSIVE ARRAY OF PROTECTIVE MECHANISMS, A NUMBER OF PATHOGENS CAN INVADE THE HUMAN BODY THROUGH THE RESPIRATORY TRACT.

What kinds/types of situations would you take orthostatic blood pressure checks? (Ch.18 4th ed. pg 433)

Orthostatic hypotension: abnormal drop in BP that occurs when a person stands after having been seated or in supine position ● Drugs (#1 reason) ○ Some induce hypovolemia which causes hypotension (diuretics) ● Immobility (fall risk patients) ● Patients on prolonged bed rest ○ Causes reduction in plasma volume, a decrease in venous tone, failure of peripheral vasoconstriction, and weakness of the skeletal muscles that support the veins and assist in returning blood to the heart ● An older patient ○ Due to decrease in systolic BP with age ● Autonomic nerve disorders

Preload/Afterload/Cardiac Output

Preload- amount the ventricles stretch at the end of diastole (aka end diastolic volume) ● Increase preload = increase stroke volume = cardiac output ● Ways to increase preload: want to do this with hypovolemic patients ○ Administer IV fluids: drop fluids into the venous system which will increase the venous blood return to the heart. So for increasing blood volume that's returning to the heart, the amount of volume that is filling the ventricles at the end of diastole is high. ○ Stimulate sympathetic nervous system (body can do naturally or give meds such as vasopressors). This causes vasoconstriction of blood vessels which will increase venous blood return to the heart. ● Ways to decrease preload: want to do this with heart failure patients ○ Administer diuretics: uses the renal system & removes extra fluid from the blood volume and the pt will urinate that out (ex. Furosemide aka lasix). This will decrease the amount of venous blood volume that is returning back to the heart. This means that the ventricles will not stretch as big. ○ Vasodilation (ex. nitroglycerin): this will decrease the amount of venous return that is returning back to the heart. Afterload - pressure that the ventricles must work against to open the semilunar valves so blood can leave the ventricle and go to the lungs or the body ● high vascular resistance = increased afterload ● Ex. the aorta has systemic vascular resistance which means there is pressure from the aorta that is keeping the aortic valve closed until the ventricle can overcome the system vascular resistance to open the aortic valve. ● Increase afterload: ○ Vasoconstriction: can affect the pulmonary & systemic vascular resistance by increasing the pressure that the ventricles must overcome to get the semilunar valves open ○ Pulmonary hypertension: increase the pulmonary vascular resistance which will increase workload on the right ventricle to overcome the pressure to open the pulmonic valve ○ High BP: increase workload of the left ventricle ○ Aortic stenosis: obstruction of out flow that is leaving the left ventricle. The aortic valve is narrowed which will increase the pressure that the left ventricle has to overcome to get the blood through the valve ● Decrease afterload: ○ Vasodilators: decrease vascular resistance & afterload Cardiac output = amount of blood the heart pumps throughout the body per minute (maintains tissue perfusion) Cardiac output = stroke volume x heart rate Stroke volume- amount of blood pumped by a ventricle with each beat ● Stroke volume is affected by 3 things: ○ Increased contractility = increased injection of blood from ventricles ○ Preload ○ Afterload

Risk factor prevention for chronic bronchitis

Prevention: quit smoking, receive annual flu vaccination and regular pneumococcal vaccinations. Avoid lung irritants like secondhand smoke, air pollution, dusts and chemical fumes.

Regurgitation (pg 690-692 5thEd.)

Regurgitation: The valve will not close all the way ● It leaks when "closed" ● Will hear a murmur of blood leaking back through when the valve should be closed Mitral Valve Regurgitation- is the incomplete closure of the mortal valve, with LV stroke volume (SV) divided between for forward SV that moves into the aorta and the regurgitation SV that moves back into the left atrium during systole a. Mitral Valve Regurgitation can result from rupture of the chordae tendineae or papillary muscles, papillary muscle dysfunction, or stretching of the valve structures caused by dilation of the LV or valve orifice. Mitral valve prolapse is a common cause of mitral valve regurgitation. i. Acute mitral valve regurgitation 1. May occur abruptly, such as with papillary muscle dysfunction after Myocardial Infarction (MI), valve perforation in (IE), or ruptured chordae tendineae in mitral valve prolapse. Acute mitral valve regurgitation is almost always symptomatic. a. Acute severe mitral valve regurgitation i. Acute volume overload increases LV preload, allowing a modest increase in LV SV. The forward SV is reduced, and the regurgitant SV leads to a rapid rise in left atrial pressure and pulmonary edema and decrease in cardiac output (CO). ii. Chronic mitral valve regurgitation 1. The hemodynamic changes associated with chronic mitral valve regurgitation occur more slowly, allowing for recruitment of compensatory mechanisms.

Restrictive respiratory disease vs. obstructive respiratory diseases

Restrictive respiratory diseases cause a decrease in lung volume, while obstructive respiratory disease causes an increase in lung volume because of trapped air in the lungs. ● Obstructive caused by airway obstruction, inflammation of the airways, and problems exhaling. ○ ex) asthma, bronchiectasis, COPD ○ Harder for patients to exhale ● Restrictive caused by poor breathing mechanics or interstitial lung disease ○ ex) obesity, scoliosis, ARDS, pneumoconioses ○ Harder for patientient to inhale -FVC1 is the volume of air that is forcibly blown out in one second, after full inspiration. -FVC (Forced Vital Capacity) is the volume of air that is forcibly blown out after full inspiration and expiration. These values are measured with a spirometry and can identify whether one has a Restrictive or Obstructive disease ● Ratio of FVC1 AND FVC determines if it is restrictive or obstructive respiratory disease. -If ratio increases=Restrictive -If ratio decreases=Obstructive - FVC1/FVC

Risk factors for deep vein thrombosis development (Slide 28 & Pages 653-654, 5th Edition)

Smoking, age, hypertension, sedentary lifestyle Venous Thrombosis (thrombophlebitis): is the presence of thrombus and the accompanying inflammatory response in the vein wall. ● Thrombus: a stationary mass of clotted blood or other formed elements that remains attached to its place of origin along the wall of a blood vessel , frequently obstructing the circulation (plural: thrombi). ● Can form in deep or superficial tissues. ● Deep vein thrombosis (DVT) is most common in lower extremities ● If left untreated it can lead to possible pulmonary embolism (PE) ● Can be asymptomatic due to lack of total occlusion of vessel ● Blood thinners (anticoagulants, i.e. heparin, warfarin) are administered for prevention RISK FACTORS ● Bed rest, immobility, spinal cord injury, acute myocardial infarction, congestive heart failure, shock & venous obstruction. Refer to Virchow's triad. ● Smoking, age, travel, sedentary lifestyle, taking oral contraceptives, diabetic, females (estrogen/birth control increases chances of blood clots) lifestyle are risk factors

Treatment of common cold, rhinosinusitis, influenza, pneumona, TB (chapter 30, pg 779, 5th edition; Chapter 22 powerpoint)

The Common Cold: Usually acute & self-limited illness in ppl who are otherwise healthy. Symptomatic treatment with rest & antipyretic drugs is usual. Antibiotics ineffective in viral infections & not recommended. - Antihistamines popular for drying nasal secretions. - Decongestant drugs constrict blood vessels in swollen nasal mucosa & reduce nasal swelling - Vitamin C has been found beneficial in shortening duration of a cold if taken before onset of the cold, but shown no true effect on incidence with general population Rhinosinusitis: treatment depends on cause & includes appropriate use of antibiotics, mucolytic agents, and symptom relief measures - Tx of bacterial infection with antibiotic/s - Intranasal glucocorticoids for inflammation - Oral & topical decongestants to promote adequate nasal drainage & congestion - Mucolytic agents to help thin secretions - Topical corticosteroids to decrease inflammation - Non-pharmacological measures: saline nasal sprays, nasal irrigation, & mist humidification Influenza: prevention through immunization (people ages > 6 months). - early diagnosis can reduce inappropriate use of antibiotics and provide opportunity for use of antiviral drug - Goal of treatment are designed to limit infection to the upper respiratory tract - Symptomatic treatment: rest, keeping warm, managing fever(antipyretics), staying well hydrated, analgesics(tylenol or ibuprofen or both), cough medication (robitussin) Pneumonias: viral vs bacterial ; inflammation of lung structures such as alveoli and bronchioles; - Antibiotics - anti inflammatory - Antipyretics Tuberculosis: bacterial; goal of treatment are to eliminate tubercle bacilli from infected person while avoiding significant drug resistance - Treatment of active TB requires multiple drugs for a relative long period; to avoid drug resistance - Primary drugs used: INH, rifampin, pyrazinamide, ethambutol, and streptomycin

Endotoxins, adhesion factors, exotoxins, evasive factors (Ch 10, pg 225, 5th edition)

These are categorical groups of virulence factors, which are substances or products generated by infectious agents that enhance their ability to cause disease Exotoxins: proteins released from bacterial cells during growth that may damage cells ● Commonly described as neurotoxins, enterotoxins, and cytotoxins exotoxins ● Exotoxins are superantigens that elicit a response through interaction with the innate and adaptive immune system to induce an inflammatory response either locally or systemically ● Bacterial exotoxins can modify cellular constituents → cell death or dysfunction Endotoxins: complex molecules composed of lipid and polysaccharides found in the cell wall of gram-negative bacteria ● Lipid portion of the endotoxin confers the toxic properties to the molecule ● Small amounts in the circulatory system can induce: clotting, bleeding, inflammation, hypotension, and fever → endotoxic shock ● At high levels can precipitate septic shock , disseminated intravascular coagulation, and acute respiratory distress syndrome Adhesion factors: positive interaction between the surface of the host cell and the infectious agent. How the interaction between microorganism and human progress into an infection or disease → through colonization ● Site specific, cell specific, nonspecific ● Receptor → site where microorganism adheres ● adhesion → substance that binds to the receptor Evasive factors: factors produced by the microorganism that enhance virulence by evading various components of the host's immune system

Renin-Angiotensin Aldosterone System (textbook 4th ed. P 424)

the trigger: low BP or low Volume Goals: is to increase your BP, A mechanism of BP Regulation used to regulate arterial pressure. The mechanism used to regulate the arterial pressure depends on whether short term or long term adaptation is needed. For short term adaptation and long term, there is a hormonal mechanism known as the Renin-Angiotensin Aldosterone System (humoral mechanism) Key players: Organs and hormones in play. Need to understand the effects of the hormone. 1. Angiotensinogen 2. Renin/Kidney a. Def: synthesized and stored by the juxtaglomerular cells of the kidney b. Released in response to 1. An increase in the sympathetic nervous system activity (b/c you want to increase BP fight/flight) or 2. decrease in blood volume, BP, c. Kidney releases Renin into the bloodstream d. Convert Angiotensinogen → Angiotensin I 3. Angiotensin I turns into Angiotensin II a. It turns into angiotensin II by the Lungs, which has an angiotensin-converting enzyme converting enzyme 4. Angiotensin II (hormone) a. Function 1: acts as a strong vasoconstrictor particularly for arterioles (short term adaptation) b. Function 2: stimulates Aldosterone (hormone) increasing salt and water retention by the kidneys 5. Adrenal Cortex/ Aldosterone a. Aldosterone is stored in the adrenal cortex b. Aldosterone tells the kidneys to resorb their salt and water (theraby increase volume) → increases BP Video: https://www.youtube.com/watch?v=tiirLoa5jK0&t=254s

Type I reactions signs and symptoms with intervention

· TYPE 1: IgE-mediated reactions, Allergic reactions · Antigens = allergens; cells central to Type 1: type 2 helper cells, granule-containing cells mast and/or basophils · Primary or immediate phase § Vasodilation, vascular leakage, and smooth muscle contraction § Occurs within 5-30 minutes of exposure to antigen § Subsides within 60 minutes · Secondary or late-phase § More intense infiltration of tissues with eosinophils an other acute and chronic inflammatory cells, as well as tissue destruction in the form of epithelial damage § Occurs about 2-8 hours later and lasts for several days § Plays a protective role in the control of parasitic intestinal infections · Local (Atopic) Reactions o Antigen confined to a particular site by virtue of exposure o Atopic disorders = commonly allergic to more than one (often many) environmental allergens o Most common: urticaria (hive), allergic rhinitis (hay fever), atopic dermatitis, food allergies, some forms of asthma o Patients with atopic allergic conditions : tend to have high serum levels of IgE and increased numbers of basophils and mast cells o Tend to manifest during childhood o Food Allergies § Primary target of food allergy: skin, GI tract, respiratory system, or any combination of the three § Most common causes: milk, eggs, peanuts, tree nuts, fish, shellfish · Foods most commonly responsible for anaphylaxis: peanuts, tree nuts, shellfish · Anaphylaxis may occur when certain food is followed by exercise § Acute reactions: hives and anaphylaxis § Chronic reactions: asthma, atopic dermatitis, GI disorders § MANAGEMENT: · Avoidance of food or foods responsible for the allergy o Allergic rhinitis § Sneezing, itching, watery discharge from the nose and eyes (rhinoconjunctivitis) § Produces nasal symptoms and also associated with other chronic airway disorders such as sinusitis and bronchial asthma § Severe attacks: malaise (general discomfort), fatigue, muscle soreness from sneezing § Fever is absent § Sinus obstruction may cause headache § Symptoms that become worse at night = household allergen § Symptoms that become improve or disappear on weekends = occupational exposure § MANAGEMENT: · Avoidance of offending agent · Oral antihistamines and oral/topical decongestants · Intranasal corticosteroids effective when used appropriately · Intranasal cromolyn useful when administered before expected contact with offending allergen · Specific immunotherapy program ("allergy shots") when symptoms are particularly challenging · Desensitization: frequent (often weekly) injections of the offending antigens Systemic (Anaphylactic) Reactions o Life-threatening o Widespread edema, difficulty breathing, vascular shock to vasodilation o Level of severity depends on sensitization o Itching, hives, skin erythema followed by bronchospasm and respiratory distress o Vomiting, abdominal cramps, diarrhea, and laryngeal edema and obstruction o Person may go into shock and die unless effective treatment instituted o MANAGEMENT § Initial focus: establishment of a stable airway and intravenous access and administration of epinephrine § Prevention of exposure to potential triggers § Patient education on how to inject epinephrine § Patients are advised to wear or carry a medical alert bracelet, necklace, or other identification to inform medical emergency personnel of the possibility of anaphylaxis

Signs of acute inflammation and their stages

○ Acute ○ Chronic 2. Vascular Phase ○ In the vascular phase, small blood vessels adjacent to the injury dilate (vasodilatation) and blood flow to the area increases. The endothelial cells initially swell, then contract to increase the space between them, thereby increasing the permeability of the vascular barrier. This process is regulated by chemical mediators ○ Exudation of fluid leads to a net loss of fluid from the vascular space into the interstitial space, resulting in edema ○ The formation of increased tissue fluid acts as a medium for which inflammatory proteins (such as complement and immunoglobulins) can migrate through. It may also help to remove pathogens and cell debris in the area through lymphatic drainage. 3. Cellular Phase ○ The predominant cell of acute inflammation is the neutrophil. They are attracted to the site of injury by the presence of chemotaxins, the mediators released into the blood immediately after the insult. ○ The migration of neutrophils occurs in four stages ➢ Margination - cells line up against the endothelium ➢ Rolling - close contact with and roll along the endothelium ➢ Adhesion - connecting to the endothelial wall ➢ Emigration - cells move through the vessel wall to the affected area ○ Once in the region, neutrophils recognise the foreign body and begin phagocytosis, the process whereby the pathogen is engulfed and contained with a phagosome. The phagosome is then destroyed via oxygen-independent (e.g. lysozymes) or oxygen-dependent (e.g. free radical formation) mechanisms Wound Healing ● 1. Inflammatory Phase: (at time of injury) main objective to increase blood flow and oxygen delivery to wound site, allows for greater energy production and faster removal of dead tissue; blood vessels in the wound contract and a clot is formed → then blood vessels dilate to allow in essential cells (i.e. antibodies, WBCs, growth factors, enzymes and nutrients) to wound ○ Cardinal Signs of Inflammation: redness, swelling, heat, pain, loss of function ○ Know acute and chronic inflammation! ● 2. Proliferative Phase: (2/3 days - 3 weeks) wound is rebuilt with collagen and extracellular matrix (ECM) by fibroblasts, in the form of a scar which is weaker tissue, sets the stage for the rest of the healing process ○ Signs: less warmth, tenderness decreases, pain felt with tissue resistance/stretch of tissue ● 3. Maturational or Remodeling Phase: (3 weeks - 2 years) final and longest phase, collagen synthesis is ongoing in order to strengthen tissue, remodeling occurs as wound continues to contract (if needed as in second intention) and fibers are being reorganized ○ Signs: improves range of motion and strength

Signs and Symptoms (Rhinovirus, parainfluenza, RSV, coronaviruses, adenoviruses, and human metapneumovirus)

● "Common colds" ○ Usually begins with a feeling of dryness and stuffiness, affecting the nasopharynx ○ Followed by rhinitis (excessive production of nasal secretion and watery eyes) ○ Often there is postnasal dripping causing irritation in the pharynx and larynx and leading to sore throat and hoarseness. ○ Affected person may experience headache and generalized malaise (general discomfort: fatigue, diffuse pain, or lack of interest) ○ Severe cases people experience chills, fevers, and exhaustion ● Rhinosinusitis ○ Acute viral Rhinosinusitis; ■ Facial pain, headache, purulent nasal discharge, decrease sense of smell, and fever ● Usually resolve within 5 to 7 days without medical treatment ● People who are immunocompromised often present with fever of unknown origin, rhinorrhea, or facial edema. ● Chronic Rhinosinusitis ○ Sinus pressure with nasal congestion ■ Symptoms may persists for years with periods of greater severity than others ■ Person may complain of a headache that is dull and constant Influenza ● Influenza A or B results in abrupt onset of fever and chills, rigors, malaise, muscle aching, headache, profuse, watery nasal discharge, nonproductive cough (dry cough), and sore throat ○ Rapid onset may occur sometimes as 1 to 2 minutes of profound malaise ● Influenza C symptoms are similar to common cold ● Influenza rated deaths can result from pneumonia and exacerbationg os cardiopulmonary conditions and other disease ● Reye syndrome (fatty liver with encephalitis) is a rare complication of influenza, particularly in youn children who have been given aspirin as an antipyretic agent Viral Pneumonia ● Occurs as a complication of influenza, most frequently in older adults or in people with cardiopulmonary disease but has been reported in healthy people ● Develops 1 day after onset of influenza and characterized by rapid progression of symptoms ● Adults with comborbidities have exacerbations ● Clinical course of influenza pneumonia progresses rapidly which can cause hypoxemia and death within a few days of onset. ○ Survivors often develop diffuse pulmonary fibrosis.

Bradykinin effect on the person's signs and symptoms

● Bradykinin: product of the liver, helps in coagulation system, causes increased capillary permeability and pain ○ Increases cap permeability and pain

Know Cold Viruses and other common viral infections

● Common Cold (Rhinoviruses): occur in early fall and late spring in persons between ages 5-40 ● Parainfluenza viruses: occur in children younger than 3 ● Respiratory syncytial virus (RSV): occur in winter and spring in children younger than 3, as we grow older, we gain an immunity to RSV ● Infections from RSV and parainfluenza viruses are most common and severe in infants and children younger than 6 years ● Coronaviruses and adenoviruses: occur in winter and spring ● Rhinosinusitis (Sinusitis): ● Infection or allergy obstructs sinus drainage (inflammation of the rhino "nose") ● Acute: facial pain, headache, purulent nasal discharge, decreased sense of smell, fever ● Chronic: nasal obstruction, fullness in the ears, postnasal drip, hoarseness, chronic cough, loss of taste and smell, unpleasant breath, headache ● Influenza: ○ In the US, approximately 36,000 persons die each year of influenza-related illness. ○ Transmission is by aerosol (3+ particles) or direct contact ○ Upper respiratory infection (rhinotracheitis): like a common cold with profound malaise (feel bad) ○ Viral pneumonia: fever, tachypnea, tachycardia, cyanosis, hypotension ○ Respiratory viral infection followed by a bacterial infection

Histamine effect on the person's signs and symptoms

● Histamine: principal mediator of immediate or acute inflammatory response, produce changes in blood vessel tone, highest concentration in mast cells near blood vessels, also in circulating platelets and basophils → released in response to trauma and immune rxns involving binding of IgE antibodies to basophils and mast cells → binds to H1 receptor in endothelial cells → increase dilation of arterioles and permeability of venules ○ inflammation

Parasitic infections

● Definition: Any organism that derives benefits from its biologic relationship with another organism is a parasite. In microbiology, it means members of the animal kingdom that infect and cause disease in other animals that includes protozoa, helminths, and arthropods. ● Types of parasites: ○ Protozoa: Malaria, amoebic dysentery (diarrhea), giardiasis ■ Unicellular; have nucleus and organelles; can be seuxal or asexual ■ Can be passed directly from host to host through sexual contact, indirectly through contaminated water or food ○ Helminths: roundworms, tapeworms, flukes ■ Wormlike parasites ■ Transmission occurs primarily through the ingestion of fertilized eggs or the penetration of infectious larval stages through the skin ○ Arthropods: ticks, mosquitos, mites, lice, fleas ■ Infest external body surfaces and cause localized tissue damage or inflammation secondary to the bite or burrowing action ■ Transmission occurs directly by contact with immature or mature forms of the arthropod or its eggs found on the infested host or the host's clothing, bedding, or grooming articles such as combs and brushes

HIV positive blood tests—which test Is used for staging and severity of disease?

● Diagnostic Methods: i. HIV ANTIBODY TEST: 1. Most accurate and inexpensive method for identifying HIV infection 2. HIV antibody test procedure: a. Enzyme immunoassay (EIA) aka enzyme-linked immunosorbent assay (ELISA) i. Detects antibodies produced in response to HIV infection ii. Antibodies to HIV in blood sample bind to HIV antigens in the test material b. Western Blot Assay i. Performed if EIA is positive ii. In a false positive EIA, western blot assay can also identify the person as uninfected iii. More sensitive assay: looks for presence of antibodies to specific viral antigens ii. Polymerase chain reaction (PCR) 1. Technique for detecting HIV DNA 2. Detects the presence of the virus rather than the antibody to the virus 3. Useful in diagnosing HIV infection in infants born to infected mothers 4. Useful in determining acute HIV infection as the antibody tests are negative in early infection ● CD4 T-cells are destroyed by HIV ● Good indicator of how well immune system is working and risk of opportunistic infections. ● CD4 T-cell count= objective measure of immunosuppression i. distinguishes three stages of HIV infection based on CD4 T-cell counts. ii. Good health 500-1,600 cells/mm^3.

Know adult parameters for fever and routes for parameters (Pages 369-373, 5th Edition)

● Fever represents an increase in body temperature that results from a cytokine-induced increase in the set point of the thermostatic center in the hypothalamus ● Routes for antipyretics administration - oral, IV, rectal, topical (cold compress) ● Fever is a nonspecific response that is mediated by endogenous pyrogens released from host cells in response to infectious or noninfectious disorders. ○ Indicates a disease state and possible need for medical treatment. ○ Prostaglandins formed in the hypothalamus alter the "setpoint". This is why Aspirin and NSAIDs work as antipyretics, they block formation of prostaglandins. Manifestations of Fever ● Anorexia, fatigue, malaise - not eating and resting conserves energy the body needs to fight off invaders. ● Chills from the body's attempt to warm itself - triggers shivering. When new setpoint is reached, chills cease. Temperature control takes place in the hypothalamus ● Low grade fever in adults typically occurs when body temperature increases to >100.4°F (38°C). ● High grade fever happens when your body temperature is >103°F (39.4°C) or above. ● Maximum temp is usually = 105°F (41°C) ● You can take a temperature using the mouth (oral), anus (rectal), armpit (axillary), or ear (tympanic). But the temperature readings vary depending on which one you use, and you need an accurate body temperature to determine if a fever is present. Know the differences in the point values for the different temperature measuring methods. ○ The average normal oral temperature is 98.6°F (37°C). ○ A rectal temperature (most accurate) is 0.5°F (0.3°C) to 1°F (0.6°C) higher than an oral temperature. ○ An ear (tympanic) temperature is 0.5°F (0.3°C) to 1°F (0.6°C) higher than an oral temperature. ○ An armpit (axillary) temperature is usually 0.5°F (0.3°C) to 1°F (0.6°C) lower than an oral temperature. ○ A forehead (temporal) scanner is usually 0.5°F (0.3°C) to 1°F (0.6°C) lower than an oral temperature. Routes for antipyretics - oral, IV, rectal

Signs & Symptoms of Heart Failure

● Fluid retention & edema ○ nocturia - nightly increase in urine output (early sign) ● Respiratory manifestation ○ dyspnea ○ Congestion of pulmonary circulation ○ exertional dyspnea ● Fatigue, weakness, & cognitive impairment ○ Diminished output from left ventricle ○ Progresses through the day ○ Anxiety, restlessness, insomnia ● Cachexia & malnutrition ● Cyanosis ○ desaturated hemoglobin ○ Agitation is an early sign of cyanosis ● Arrhythmias & sudden cardiac death Right - sided HF "SWELLING" Swelling of leg, hands, liver Weight gain Edema (pitting) Large neck vein (jugular vein distention) Lethargic Irregular HR (afib) Nocturia (lying down allows fluid to go to the kidneys) Girth (abdomen increases in size, breathing issues, anorexia, nausea) Left - sided HF "DROWNING" Dyspnea Rales (crackles) Orthopnea (sitting up to breathe especially @ night) Weakness Nocturnal paroxysmal dyspnea Increased HR Nagging cough - blood tinged sputum Gaining weight (2-3 pounds in a day or 5/week)

Differences and possible signs and symptoms on a patient and possible causes due to HIV:

● Formation of antigen-antibody complex (Type III Hypersensitivity): free floating antigen + antibody → circulating immune complex → immune complexes deposit on walls of blood vessels & activates complement → increase vascular permeability & recruit phagocytic cells → all of which can promote local tissue damage and edema ○ S/S: urticaria, patchy or generalized rash, extensive edema (usually of the face, neck, and joints), and fever ○ Responsible for the tissue injury seen in some forms of glomerulonephritis & vascular rejection of organ grafts ● Cytokine release by sensitized T-cells (Type IV Hypersensitivity): cytokines secreted by T helper cells activate macrophages and cytotoxic T cells → sensitized T cells attack any cell with the virus regardless if virus is harmful or not → body's immune system causes more harm than the virus ○ Occurs in allergic contact dermatitis (Example: POISON OAK, hypersensitivity pneumonitis "farmer's lung" ● Ig-E mediated mast cell degranulation (type I hypersensitivity): ○ i. Occur rapidly (w/I mins) ○ ii. Involved in inflammation, allergic responses and to combat parasitic infections ○ iii. Binds to mast cells and triggers release of histamine and other mediators important in inflammation and allergies ○ iv. Signs and symptoms: urticaria, rhinitis (hay fever), bronchial asthma, Vasodilation, smooth muscle contraction, ● Formation of antibodies against cell surface antigens (Type II Hypersensitivity): ○ IgM or IgG bind antigens to cell surface ○ Ex. mismatched blood transfusion reaction or hemolytic disease in newborns due to ABO or Rh incompatibility ○ s/s: transient anemia, leukopenia, thrombocytopenia

Prevention of Cold Viruses and other common viral infections

● Hand hygiene ○ Wash your hands ○ Avoid touching your face ○ Avoid stress and get lots of rest (boost immune system) ● Facial coverings ● Vaccinations ● Geriatric population should get vaccines because they're the most vulnerable and should be vaccinated to avoid death

If a fever is labile (unstable), when would you want to recheck temperatures?

● If a patient's fever is intermittent, need to check for signs/symptoms of fever & recheck temperature: ○ Nonspecific complaints such as mild headache and fatigue ○ A chill, during which the temperature rises ○ A flush, during which the skin becomes warm and flushed ○ A defervescence stage, which is marked by the initiation of sweating ○ Check every 1-4 hours depending on situation ■ Depending if pt is exhibiting acute change in symptoms ■ Reassess a patient if they have chills or a fever.

HIV patients' opportunistic infections

● If a person is HIV positive their immune system becomes severely compromised which is the decline in CD4+ T cells. Due to this decline, HIV positive people are at risk of catching infections more easily making it an opportunistic infection. ○ Example infection: bacterial pneumonia (PCP) (TB), fungus,viral,protozoa (toxoplasmosis) leading to respiratory, GI,nervous system issues, oral thrush, esophagitis, skin staph infection

Assessment of patient risk for contraction of bacterial, viral, fungal infections

● Immunocompromised pts ● Cancer pts ● Transplant pts ● Kids, geriatric ● HIV pts

What are Signs and Symptoms of lower ejection fraction

● Increase sodium and water retention by the kidney ● Resultant increase in vascular volume ● Fluid retention ● Shortness of breath or inability to exercise. ● Swelling of the feet and lower legs. ● Fatigue and weakness. ● Rapid, forceful, uncomfortable or obviously irregular heart beat. ● Abdominal discomfort such as swelling, pain or nausea. ● Mental confusion.

Antiviral medications in viral infections, how they work and what they do

● Interfere with the viral replication process ● This can decrease the frequency of occurences

Why do injured cells swell?

● Lecture slide info (Chapter 20): ○ Decreased Perfusion limits cellular ATP ○ When there is no oxygen for aerobic metabolism Pyruvate is converted to Lactic Acid (causing metabolic Acidosis) ■ Builds up in the cellular and extracellular space ○ No ATP = No Cellular Function ■ Na/K ATPase pump stop working with no ATP ● NA in cell, loss of potassium ● Cell Swelling → Cell destruction due to Lysosomes Cells swell due to interleukines, WBCs, histamines, Inflammatory cascade ● Textbook info: ○ Cellular swelling occurs with impairment of the energy-dependent Na/K- ATPase pump (Na→ out, K→ in), usually as a result of Hypoxic Cell Injury. ○ With impaired function of the pump, intracellular potassium levels decrease,and sodium and water accumulate in the cell. ○ Reversible if oxygenation is restored. ○ However, if the oxygen supply is not restored there is continuous loss of enzymes, proteins, and RNA through the hyperpermeable cell membrane. Injury to the lysosomal membranes results in the leakage of destructive lysosomal enzymes into the cytoplasm and enzymatic digestion of cell components. ○ Leakage of intracellular fluids can be measured in lab tests and a great indicator of cell injury/death. ○ ATPase pump has impaired function with injury to cells and thus K+ decreases intracellularly and Na+ and water increase intracellularly causing it to swell

Signs of a local and systemic infection

● Local infection - pain, heat , redness (erythema), swelling (edema), site-dependent loss of function, purulent exudate drainage Ex. wound, URI, localized wound can turn into systemic infections ● Systemic infection - fever, malaise, fatigue, chills, confusion, nausea, vomiting, sepsis, increased HR ○ When bacteria/virus propagate through the blood -Inoculation of a wound that goes sepsis is an example of a localized infection going systemic.

DASH diet and methods to reduce hypertension (Ch.18 4th ed. pg 429)

● Methods to reduce hypertension: ○ DASH diet: Dietary Approaches to Stop Hypertension ■ Reduce sodium ■ Eat more fruits & vegetables ■ Low-fat dairy ■ Whole grains ■ Some fish, poultry and legumes ■ Small amount of nuts and seeds ○ Weight reduction in persons who are overweight or obese ○ Regular physical activity ○ Reduction of dietary sodium intake ○ Limitation of alcohol intake ■ Males: <2 drinks/day ■ Females and persons of lighter weight: <1 drink/day

Assessment of Heart failure patient

● Methods used to diagnose/assess include risk factor assessment, history/physical examination, lab studies, EKG, chest radiography, and echocardiography. - History should include info related to dyspnea, cough, nocturia, general fatigue, other S/S of heart failure - Physical exam includes assessment of HR, heart sounds, BP, jugular veins for venous congestion, lungs for pulmonary congestion, and low extremities for edema - Lab tests diagnose anemia, electrolyte imbalances, and detect signs of liver congestion ● Invasive hemodynamic monitoring may be used for assessment in acute, life threatening episodes of HF - Methods include: 1. CVP (central venous pressure) - reflects amount of blood returning to heart, decreases in hypovolemia and increases in right-sided HF → changes that occur in CVP over time usually more significant than a single reading 2. Pulmonary artery pressure monitoring 3. Thermodilution measurements of CO 4. Intra-arterial measurements of BP

Modifiable vs. non-modifiable risk factors for Primary HTN (Ch.18 4th ed. pg 426)

● Modifiable: lifestyle ○ Diet (ex. DASH diet) ○ Physical activity ○ Stress management ○ Alcohol ● Non-modifiable: ○ Family hx ○ Genetics ○ Race ○ Age ○ Metabolic syndrome ○ Insulin resistance: increases chances that a person will have high cholesterol which causes pressure on arteries and veins → HTN

Nitric Oxide effect on the person's signs and symptoms

● Nitric Oxide: relaxation of vascular smooth muscle, antagonism of platelet adhesion, aggregation, and degranulation, regulator of leukocyte recruitment, endogenous NO reduces cellular phase of inflammation, impaired NO production by endothelial cells → inflammatory changes that can lead atherosclerosis. ○ smooth muscle relaxation

Signs/Symptoms—> Abdominal Aortic Aneurysm

● Pain may be present and varies from mild mid abdominal or lumbar discomfort to severe abdominal and back pain ● Most abdominal aneurysms are asymptomatic ● As the aneurysm expands, it may compress the lumbar nerve roots, causing lower back pain that radiates to the posterior aspects of the legs ● Unruptured aneurysms are generally asymptomatic and are often diagnosed incidentally during clinical examination. ● Because an aneurysm is of arterial origin, a pulsating mass may provide the rst evidence of the disorder. Typically, aneurysms larger than 4 cm are palpable. ● Calcification, which frequently exists on the walls of the aneurysm, may be detected during abdominal radiographic examination ● The aneurysm may extend to and impinge on the renal, iliac, or mesenteric arteries, or to the vertebral arteries that supply the spinal chord. ● May cause erosion of vertebrae ● Stasis of blood favors thrombus formation along the wall of the vessel, and peripheral emboli may develop, causing symptomatic arterial insufficiency

Patient Education —> Abdominal Aortic Aneurysm (know how you would educate pt on this, when you should go to ER, when you should stay home)

● Pain may be present: i. mild abdominal or lumbar discomfort ii. Severe abdominal and back pain iii. As aneurysm expands: lower back pain that radiates to the posterior aspect of the legs ● Palpable pulsatile mass may provide the first evidence of disorder ● Signs and symptoms depend on the size and location ● Unruptured aneurysms are generally asymptomatic often diagnosed during clinical examination ● Control hypercholesterolemia and high blood pressure should be controlled ● Discontinue smoking

Signs and symptoms of Pleuritic chest pain

● Pleuritic chest pain is characterized by sudden and intense sharp (ABRUPT!), stabbing, or burning pain in the chest when inhaling and exhaling. It is exacerbated by deep breathing, coughing, sneezing, or laughing. ● When pleuritic inflammation occurs near the diaphragm, pain can be referred to the shoulder ● Usually made worse by chest movements, breathing, coughing, can be confused with broken rib symptoms ● Small tidal volumes ● Rapid breathing ● Reflex splinting of the chest may occur

Signs and time frame for possible blood transfusion reactions Type II antibody mediated hypersensitivity disorder.

● S/S of possible blood transfusion reactions a. Fever, chills, pruritus, or urticaria b. Severe SOB, red urine, high fever, or loss of consciousness c. O2 drop d. Temperature drop e. Lack of consciousness f. Pallor g. Shortness of breath h. Agitation due to impending hypoxia ● Time frame for possible blood transfusion a. Acute: 24 hours or less b. Delayed: 4 - 8 days c. Mismatch reaction happens quickly ( 5 mins mentioned during review) ● Type II (antibody-mediated) hypersensitivity reactions are mediated by IgG or IgM antibodies directed against target antigens on cell surfaces or in connective tissues. Three different antibody-mediated mechanisms are involved: a. Complement-and antibody- mediated cell destruction b. Complement - and antibody-mediated inflammation c. Antibody-mediated cellular dysfunction

Serotonin effect on the person's signs and symptoms

● Serotonin: produce changes in blood vessel tone, similar effects as histamine, stored in platelet granules and released during platelet aggregation ○ controls bleeding

Know the differences: serotonin, histamine, bradykinin, nitric oxide and their effect on the person's signs and symptoms

● Serotonin: produce changes in blood vessel tone,similar effects as histamine, stored in platelet granules and released during platelet aggregation ○ controls bleeding ● Histamine: principal mediator of immediate or acute inflammatory response, produce changes in blood vessel tone, highest concentration in mast cells near blood vessels, also in circulating platelets and basophils → released in response to trauma and immune rxns involving binding of IgE antibodies to basophils and mast cells → binds to H1 receptor in endothelial cells → increase dilation of arterioles and permeability of venules ○ inflammation ● Bradykinin: product of the liver, helps in coagulation system, causes increased capillary permeability and pain ○ Increases cap permeability and pain ● Nitric Oxide: relaxation of vascular smooth muscle, antagonism of platelet adhesion, aggregation, and degranulation, regulator of leukocyte recruitment, endogenous NO reduces cellular phase of inflammation, impaired NO production by endothelial cells → inflammatory changes that can lead atherosclerosis. ○ smooth muscle relaxation

Transmission (5th edition, pg. 780) "Common Viruses" (Rhinovirus, parainfluenza, RSV, coronaviruses, adenoviruses, and human metapneumovirus) :

● Spread rapidly from person to person ● Children are the major reservoir of cold viruses, often acquiring a new virus from another child in school or daycare. ● Fingers are the greatest source of spread, the nasal mucosa and conjunctival surface of the eyes are the most common portals of entry of the virus ● Last about 7 days with an incubation period of 2 day. ● Aerosol spread of colds and and sneezing is less important than the spread through direct mucous membrane contact by fingers picking up the virus from contaminated surfaces and carrying it to the nasal membranes and eyes Influenza: ● Incubation period is 1 to 4 days, people become infectious starting day 1 before their symptoms begin and remain infectious approximately 1 week after illness onset ● Viral shedding can continue for approximately 3 weeks. ● Inhalation of droplet nuclei rath than touching contaminated objects ● Most infected people develop symptoms of disease, increasing the spread of infectious droplets ● Young Children most likely to become infected and spread the infection Viral Pneumonia: ● Viruses that cause pneumonia is due to droplet of fluid after someone sneezes or coughs ● Viral pneumonia can also occur by touching surfaces and then a portal of entry (mouth, nose) ● Common viral causes of community acquired pneumonia (infections from organisms found in the community rather than in the hospital or nursing home) include the influenza virus, RSV, adenovirus, and parainfluenza virus. ● Hospital acquired pneumonia is defined as a lower resp. Tract infection that was not present or incubating on admission to the hospital ○ Pneumonia 2nd most common cause of hospital acquired infection ○ Most of these infections are bacterial though

Ejection Fraction (EF)

● Stroke volume /Diastolic volume= ejection fraction ○ represents the fraction or percentage of the diastolic volume that is ejected from the heart during systole ● One of the signs of (usually systolic) heart failure is a decrease in the ejection fraction, which reflects the diminished function of the left ventricle.

Know normal vs abnormal values for ejection fraction

● Systolic heart failure ○ Reduced EF <40% ● normal : 60-70% ● Stroke volume / Diastolic volume = Ejection Fraction

Pathophysiology behind tension pneumothorax

● Tension Pneumothorax: Occurs when the intrapleural pressure exceeds atmospheric pressure ● Pathophysiology: ○ The visceral pleura(attached to lungs) and parietal pleura (attached to the chest wall) and separated by a small amount of serous fluid and this space is called the intrapleural space ○ In a pneumothorax, air collects in the intrapleural space that causes the lung to collapse ○ Really good video: https://www.youtube.com/watch?v=0AlGjHNPTzk&t=928s ● Increased pressure within pleural cavity impairs both respiratory and cardiac function ● Seen most often with traumatic pneumothoraces ● Can result from barotrauma caused by mechanical ventilation ○ Due to high tidal volume on people on the ventilator ● Can sometimes be correlated to traumatic pneumothorax which is a result of direct injury to the chest or major airways ● Will have deviated trachea or flail chest 1. chest or respiratory injury lets air enter but cannot leave pleural space 2. pressure increases and heart and vessels are compressed 3. mediastinal structures and trachea shifts to opposite side of chest 4. unaffected side is compressed 5. decrease in venous return to heart and decreased cardiac output

What causes vasculitides/vasculitis? (Ch.18 4th ed. pg 411, 414)

● Vasculitides: group of vascular disorders that cause inflammatory injury and necrosis of the blood vessel wall (usually autoimmune) ● Vasculitis: inflammation of a blood vessel/arteries ● Causes: Immune system attacks its own blood vessels → scarring/thickening of blood vessels → narrows blood vessels → decreased blood flow to other parts of the body ○ Direct invasion of vascular wall by infectious agent ○ Immune-mediated inflammation (Example: Autoimmune diseases) ○ Video (Only need to watch first 2 minutes) ● Triggering event is unknown

Sodium/Water Retention

❖ Causes high blood pressure ❖ Sodium and water are joined at the hip - if you have an increase of sodium in your system, you need to have increase in water intake to balance ❖ Approximately 100mL of water per 100 calories metabolized needed to maintain balance ❖ The amount of water and its effect on sodium concentration in the ECF serves to regulate the distribution of fluid between the ICF and ECF ❖ Sodium's primary job = regulate ECF volume ❖ ANP (atrial natriuretic peptide) released from cells in the atria of the heart = stimulates kidneys to excrete sodium in order to pull out more water. ❖ Renin Pathway:

Cardiac Remodeling

❖ Remodeling is one of the principal mechanisms of compensation for work overload ❖ Remodeling includes: ➢ Symmetric hypertrophy: proportionate increase in muscle length and width ■ Athletes ■ Replicates typical heart function ➢ Concentric hypertrophy: increase in wall thickness ■ Hypertension ■ Pressure overload occurs = increase wall stress = replication of myofibrils, thickening of myocytes = concentric hypertrophy ● Leads to: ◆ Ischemia ➢ Eccentric hypertrophy: disproportionate increase in muscle length/decrease in wall thickness and ventricular dilation ■ DCM: Dilated Cardiomyopathy ■ Ventricular volume overload = increase wall stress = replication of myofibrils, elongation of myocytes = eccentric hypertrophy ● Leads to: ◆ Ventricular wall thickening ◆ Increased diastolic volume/ increased wall tension ❖ Cardiac Dyssynchrony: Fibrosis and remodeling may lead to electrical conduction abnormalities in which the heart contracts in an uncoordinated manner ➢ Causes reduced systolic heart function


Kaugnay na mga set ng pag-aaral

Audit Ch 16 - operations & completing the audit

View Set

Chapter 6- Assessing Mental Status and Substance Abuse PrepU

View Set

PrepU Ch. 13: Blended Competencies

View Set

ITIL 4 Foundation Practice Exam 5

View Set